You are on page 1of 42

TEÓRICO d.

La cardiomiopatía de Tako Tsubo está


asociada a obstrucción coronaria
Un paciente con Taquicardia Ventricular
(observada en el cateterismo cardiaco)
monomórfica que se encuentra consciente y
hemodinámicamente estable. Tras la e. Es raro la presencia de arritmias
administración de oxígeno y canalización de vía ventriculares
ev. es necesario:
Paciente de 45 años cursa con sensación de
a. Administrar 1mg de Adrenalina palpitaciones desde hace 3 meses, episódicas,
nota que inicia con disnea a moderados
b. Administrar una dosis de carga de 150 mg
esfuerzos desde hace 1 mes.
de Amiodarona
Al examen físico: PA: 110/40 mmHg FC: 88 lpm.
c. Infusión de Adrenalina
Pulmones: MV pasa bien en ACP. CV:impulso
d. Cardioversión con 100 Joules ventricular izquierdo en 5to espacio intercostal
izquierdo fuera de la línea medio clavicular,
El soplo de Graham Steel, se presenta en: impulso ventricular derecho negativo, ruidos
cardiacos totalmente arrítmicos, pulso
a. Es diastólico se presenta en
deficitario, primer ruido disminuido, soplo
Insuficiencia mitral
sistólico en 5to EICI LMC II/VI irradiado a axila
b. Es sistólico se presenta en estenosis (INSUFICIENCIA MITRAL), soplo diastólico en
mitral foco mitral II/VI.Abdomen blando depresible, no
visceromegalia, no edemas. Indique la
c. Es diastólico se presenta en respuesta correcta.
insuficiencia pulmonar
5to espacio intercostal LMC ⇒ Foco mitral
d. Es holosistólico se presenta en
estenosis aórtica a. Según lo descrito en el examen físico es
probable que haya crecimiento
e. Es diastólico se presenta en ventricular derecho.
insuficiencia Aórtica
b. El soplo continuo descrito sugiere
Una arritmia en la que tras cada onda P, tiene presencia de ventana aortopulmonar
lugar un retraso progresivo de la conducción de
los impulsos eléctricos a través del nódulo AV c. El diagnostico etiológico del cuadro
hasta que queda bloqueada por completo dicha clínico probablemente sea una
conducción, se denomina: cardiomiopatía.

a. Bloqueo AV de primer grado d. El examen físico sugiere enfermedad


valvular regurgitante asociada a lesión
b. Bloqueo AV de segundo grado tipo I estenótica.

c. Bloqueo AV de segundo grado tipo II e. El primer ruido disminuido sugiere


flexibilidad importante de los velos de la
d. Bloqueo AV de tercer grado
válvula comprometida
Con respecto a cardiomiopatías dilatadas
El cuarto ruido cardíaco se produce por:
señalar lo correcto:
a. Por contracción auricular frente a un
a. Son de causa isquémica (coronaria)
ventrículo con distensibilidad
b. Presentan principalmente disfunción disminuida
diastólica
b. Cierre de las válvulas sigmoideas
c. La cardiomiopatía periparto
c. Cierre de las válvulas atrioventriculares
mayormente es de curso reversible
d. Retardo de cierre de una de las válvulas
sigmoideas
e. Contracción auricular con dilatación d. Betabloqueadores + Inhibidores de la
importante del ventrículo izquierdo Enzima Convertidora de Angiotensina
(IECAs)
Paciente de 60 años diabético de más de 10
años, mal controlado, en consulta externa se le e. Todas las anteriores
encuentra una PA de 150/90 mmHg. Requiere
Paciente femenina 69 años, acude a consultorio
tratamiento con:
de Cardiología para control. Dx. por Historia
a. Losartan 50 mg. ½ tableta al día Clínica: Insuficiencia Cardiaca Crónica
Compensada. Al estudio Ecocardiográfico:FE =
b. No requiere fármacos, solo reducción de
38%. Antecedente de IMAprevio, FA (-),HTA (+),
sal y control de glicemia
estable hemodinámicamente, sin congestión
c. Enalapril 10 mg cada 12 hs y régimen pulmonar y con función renal preservada, edema
hiposódico ⇒ Para diabéticos dar: de MMII (-). ¿De las siguientes combinaciones
IECAS (Sd nefróticos sea HT o no) de fármacos, cuál considera usted sería
inicialmente la más indicada?
d. Hidroclorotiazida 50 mg/ dia
(CONTRAINDICADO EN DIABETES) a. Levosimendán + digitálicos

e. Dieta baja en sal, bajar de peso y control b. Antagonistas de la aldosterona (FEVI


de glicemia estricto <35) + dopamina

Paciente de 45 años sin antecedentes de c. Dobutamina + Antagonistas de los


enfermedad cardiovascular ni factores de receptores de la angiotensina (ARA II)
riesgo. Acude a consulta por cefalea leve y al
d. Betabloqueadores + Inhibidores de la
examen se le registra PA de 140/90 mmHg. Se
Enzima Convertidora de Angiotensina
inicia tratamiento con:
(IECAs)
a. Bisoprolol 5 mg /día
e. Todas las anteriores
b. Captopril 25 mg sublingual y dieta baja
El diagnóstico integral de la INSUFICIENCIA
en sal
CARDIACA consiste en la búsqueda minuciosa
c. Losartan 50 mg/ día y dieta baja en sal
de la causa, motivo por el cual se requiere
d. Dieta baja en sal y controles
además de una buena historia clínica una
e. a+d
evaluación con los siguientes procedimientos
Paciente mujer de 75 años, acude a Emergencia diagnósticos que permitirán un tratamiento
con historia de disnea CF II de 3 semanas de óptimo. Marque lo CORRECTO:
evolución. Antecedente de HTA no controlada. Al
a. Electrocardiograma
examen físico: PA 180/100 mmHg, edema de
miembros inferiores (++/+++), ortopnea, b. Ecocardiografía
ingurgitación yugular, RX tórax: signos
radiográficos de congestión pulmonar, EKG: c. Test de esfuerzo
hipertrofia de VI con sobrecarga de VI.
Laboratorio: pro-BNP 2480. Ecocardiografía: d. Tomografía
FEVI 52%, hipertrofia concéntrica del VI. e. Todas
¿De las siguientes combinaciones de fármacos, Mencione las características de la definición de
cuál considera usted sería inicialmente la más insuficiencia cardíaca
indicada?
a. Reducción de Gasto Cardíaco
a. Diuréticos + Inhibidores de la Enzima progresivo
Convertidora de Angiotensina (IECAs)
b. Incremento en las presiones de llenado
b. Betabloqueadores + Inotrópicos
c. Sobreactividad neurohormonal
c. Inotrópicos + Bloqueadores de los
Receptores de Angiotensina (BRAs)
d. Presencia de inflamación, apoptosis y Paciente mujer de 60 años, ingresa a
fibrosis emergencia con dolor toráxico, PA: 130/70; FC:
88x’; Satinfrad O2: 96%; enzimas cardiacas
e. Todas
elevadas; EKG evidencia ritmo sinusal con
Varón de 45 años con antecedentes de infradesnivel del ST en DII, DIII y aVF,¿cuál de
Fibrilación auricular e hipertensión arterial bajo las siguientes acciones indicaría?
tratamiento con losartan y warfarina. ingresa a
a. Intervencionismo coronario percutáneo
emergencia por presentar fiebre, ortopnea y
en menos de 2 hs
dolor torácico atípico hace 1 semana,
presentando hoy síncope. Al ingreso con PA b. Estratificar el riesgo: escala de GRACE
90/60, FC 120 lpm, FR 28 rpm. y 60 kg de peso, se (RECOMENDADO PARA INFRA ST)
le realiza:
c. Anticoagulación con Warfarina
- Electrocardiograma de urgencia mostrando
elevación del Segmento ST difusa en d. Antiagregación plaquetaria: AAS 75
precordiales y cara inferior. mg/día

- Ecocardiografía se evidencia una efusión e. Trombolisis con Alteplase


pericárdica circunferencial con diámetro mayor
Mujer de 35 años, gestante de 4 semanas,
de 30 mm y colapso diastólico del VD.
presenta episodio de disnea súbita, siendo
- Laboratorio PCR 3, INR 2.1, leucocitosis, evaluada y diagnosticada de tromboembolia
creatinina 1.1 mg/dl, lactato 10 mg/dl, troponina I pulmonar. En este caso el manejo inicial y a largo
negativas al ingreso. plazo que usted indicará será con:

¿Diga usted cuál sería su diagnóstico más a. Enoxaparina como manejo inicial y a
probable? largo plazo (EN TEP GESTANTES hasta el
final de gestación ENOXAPARINA SC)
a) Pericarditis constrictiva
b. Enoxaparina, seguida de warfarina (NO
b) Infarto agudo de miocardio ST elevado WARFARINA EN GESTANTE)
c) Efusión pericárdica moderada de riesgo c. Heparina sódica, seguida de warfarina
alto (TB)
d. Fondaparinux, seguido de dabigatrán
d) Taponamiento pericárdico
e. Enoxaparina, seguido de apixaban
Varón de 80 años, asmático, sin factores de
riesgo cardiovascular de importancia, acude a Paciente de 68 años, con angina de pecho,
evaluación a la consulta en silla de ruedas, insuficiencia ventricular izquierda, síncope,
llevando un electrocardiograma que demuestra presión diferencial corta, soplo eyectivo. Señale
ondas Q en DII, DIII y aVF.¿Cuál de las el diagnóstico más probable de este paciente
siguientes alternativas solicitaría al paciente a. Comunicación Interauricular
para estratificar riesgo isquémico?
b. Persistencia del Ductus arterioso
a. Prueba de esfuerzo en banda sin fin
c. Insuficiencia Mitral
b. Prueba de esfuerzo en cicloergómetro
d. Estenosis Mitral
c. Test de perfusión miocárdica con
esfuerzo físico e. Estenosis Aórtica (Síncope, Angina,
Disnea)
d. Test de perfusión miocárdica con
adenosina ⇒ Contraindicación de BANCO 2021-1
adenosina: Asma bronquial
Paciente con diagnóstico de taquicardia
e. Ecocardiograma con estrés con paroxística supraventricular,
dobutamina
hemodinámicamente estable, fallan las Familiares traen a emergencia a una mujer de
maniobras vagales, la siguiente medida es: 63 años, con historia antigua de hipertensión,
diabetes mellitus 2, porque en las ultimas 24
A) Adenosina
horas esta incoherente, a la exploración se
B) Cardioversión eléctrica observa una paciente desorientada con PA:
230/160 mmHg, frecuencia respiratoria 25 por
C) Digoxina minuto, pulso 110 lpm, y T: 36.7°C. En la
auscultación pulmones hay crepitantes
D) Betabloqueantes
bibasales y la familia refiere que ha dejado de
E) Calcio antagonistas tomar los antihipertensivos. Se monitoriza a la
paciente y se insertan vías venosa y arterial. Una
Mujer de 68 años ingresa por emergencia TEM cerebral excluye hemorragias y masa
presentando disnea al reposo, PA: 140/50 mm intracerebral. ¿cuál de los siguientes es el paso
Hg , FC: 100 X ´ , Soplo cardiaco en fase más adecuado que debe a continuación?
diastólica, pulso celer ,EKG: Hipertrofia
Ventricular Izquierda, Tele Rx. C y GV: A) Observar a la enferma en una habitación
Crecimiento de cámaras izquierdas. Indique el tranquila
diagnóstico mas probable:.
B) Esperar los resultados de laboratorio antes de
A) Insuficiencia valvular aórtica decidir el tratamiento especifico

B) Estenosis valvular mitral C) Administrar Nitroglicerina en infusión EV

C) Estenosis valvular tricuspídea ● Edema pulmonar

D) Insuficiencia valvular pulmonar ○ Fármacos a usar:

E) Insuficiencia valvular tricuspídea ■ Vasodilatadores de tipo


venoso ⇒ primera opción:
Niño de 6 años con cianosis, acropaquias, nitroglicerina, nitratos
disnea, soplo cardiaco, en la radiografía de endovenosos // tbn
tórax la punta del corazón elevada. El vasodilatación coronaria //
bajan PA.
diagnóstico más probable es:
■ NUNCA DAR DIURÉTICOS
A) Coartación de aorta
■ Labetalol
B) Conducto arterioso permeable
PARA PACIENTE CON CRISIS HTA
C) Transposición de grandes vasos sin shunt
presente Fármacos endovenosos si hay daño en órgano
blanco
D) Tetralogía de Fallot
● Nitropusiato
E) Estenosis valvular aórtica
● Nitroglicerina
Niña de 4 años que presenta infecciones
Dar medicamentos por vía oral: ARA II, otros
respiratorias frecuentes, sus controles de peso y
antagonistas de ca (no nifedipino)
talla para la edad baja, soplo en maquinaria de
Gibson. ¿Qué tipo de cardiopatía congénita más
probable presenta?
D) Administrar diazóxido sódico en bolo EV
A) Conducto arterioso permeable
E) Administrar sulfato de Magnesio EV
B) Comunicación interauricular

C) Comunicación interventricular
Paciente obeso, con triglicéridos séricos
D) Coartación de aorta incrementados, síndrome metabólico y con PA de
160/100 mmHg requiere tratamiento
E) Anomalía de Ebstein
farmacológico. ¿Qué antihipertensivo será la electrocardiográfico anormal, seria catalogado
mejor indicación? como un Sincope:

A) ARA 2 + Antagonista del Calcio A) De alto riesgo

B) IECA + Hidroclorotiazida B) De Bajo riesgo

C) Antagonista del Calcio + Hidroclorotiazida ⇒ C) De riesgo intermedio


Contraindicación de diuréticos y
betabloqueadores: Síndrome metabólico. D) No clasificado

D) ARA 2 + IECA ⇒Nunca dar ARA II y IECA E) De ningún riesgo

E) ARA 2 + Hidroclorotiazida Paciente de 30 años quien es usuario de drogas


ilícitas endovenosas, se le diagnostica
ANSWER: A endocarditis infecciosa. ¿Cuál es el
microorganismo aislado con mayor frecuencia
Respecto a las cardiomiopatías señale lo
en endocarditis infecciosa de pacientes
correcto:
drogadictos intravenosos y cual la válvula
A) La cardiomiopatía hipertrófica es la principal afectada más frecuentemente en estos
causa de muerte súbita en personas menores de pacientes?
35 años
A) Estafilococos aureus - mitral
B) Las principales causas de las cardiomiopatías
B) Streptococo viridans - mitral
son isquémica (coronaria) y valvulares
C) Estafilococos epidermidis - aortica
C) Todas las cardiomiopatías dilatadas son
irreversibles D) Cándida - tricuspídea
D) En las cardiomiopatías restrictivas el E) Estafilococo aureus – tricuspídea
electrocardiograma muestra hipertrofia (debido
a la infiltración) Varón de 57 años que acude a emergencia por
opresión en el centro del pecho irradiada a la
E) En las cardiomiopatías hipertróficas se cara anterior del cuello que se presenta luego
observa una gran cardiomegalia en la jugar squash. Refiere ser hipertenso y
radiografía de tórax por la gran hipertrofia dislipidémico en tratamiento irregular con
ventricular telmisartán, amlodipino, atorvastatina y aspirina
de manera irregular. Se le diagnostica un infarto
de cara inferior y fue sometido a cateterismo
El enfoque terapéutico que mejoro sobrevida en cardiaco encontrándose obstruida la arteria
pacientes con shock cardiogénico ocasionado coronaria derecha; se le realiza angioplastia y
por un Infarto Agudo de Miocardio fue: se implanta un stent medicado. ¿Qué conducta
debe seguir al salir de alta?
A) Trombolíticos
A) Continuar tratamiento con aspirina
B) Revascularización coronaria percutánea y/o
quirúrgica B) Debe mantener ticagrelor por un año a dosis
de 90 mg cada 12 hs.
C) Balón de contra pulsación aortica
C) Atorvastatina 80 mg post cena
D) Ventilación Mecánica
D) Valsartan por 6 meses si su fracción de
E) Aspirina eyección esta conservada

E) Todas son correctas


Si en un paciente con Sincope se evidencia que Mujer de 20 años, gestante de 15 semanas, con
fue durante el ejercicio físico, presenta antecedente de síndrome antifosfolipídico,
antecedentes cardiacos, con un trazado refiere que desde hace una semana presenta
hinchazón de la pierna derecha y desde el día ANSWER: B
de hoy disnea moderada. PA 100/60mmHg, FC
Una paciente de 24 años consulta por un
100 lpm, dímero-D positivo, se difiere Rx de tórax
episodio de poliartritis migratoria y carditis. El
por gestación. Con estos datos, el tratamiento a
paciente presenta antecedentes de fiebre
iniciar y el examen que solicitaría será:
reumática a los 8 años. Se ausculta soplo de
A) Warfarina - AngioTEM estenosis mitral. Para prevenir la aparición de
nuevos episodios de fiebre reumática, se debe
B) Apixaban - Ecocardiograma
aplicar Penicilina Benzatínica 1.200.000
C) Heparina sódica - Arteriografía pulmonar unidades intramuscular profiláctica cada mes:

D) Enoxaparina - Gammagrafía pulmonar A) Al menos por 5 años

E) Alteplase - AngioTEM B) Hasta los 30 años

Hombre de negocios de 52 años, se presenta a C) Por 10 años o hasta los 40 años de edad
la oficina quejándose de la creciente fatiga y
D) Por un año
dificultad para respirar. También ha notado
recientemente que está más cómodo durmiendo E) No necesita profilaxis
con tres almohadas. Niega molestias en el pecho
o dolor pleurítico. Sus únicos medicamentos son
hidroclorotiazida 25 mg al día y atenolol 50 mg Varón 74 años, con antecedentes de enfermedad
diarios para la HIPERTENSIÓN DE 10 AÑOS DE coronaria, HTA, diabetes mellitus 2, acude por
DURACIÓN CON BUEN CONTROL. Antecedentes: disnea a moderados esfuerzos y fatiga. Examen:
Fuma 2 paquetes de cigarrillos al día, bebe PA 110/60 mmHg FC 95 lpm SatO2 97%, edema
whisky socialmente y admite dos martinis a la MMII ++, MV pasa en ACPcrépitos difusos,
hora del almuerzo cada día. Niega antecedentes ruidos cardiacos arrítmicos, 1rc cambiante no
familiares de enfermedades cardíacas. En el soplos. EKG Fibrilación auricular con respuesta
examen, su frecuencia cardíaca es de 104 ventricular controlada. Ecocardiograma fracción
latidos/min, las respiraciones son de 20 eyección 40%. ¿Cuál sería el fármaco NO
respiraciones/min, y la presión arterial es de indicado para este paciente?
134/84 mm Hg. NO HAY CAMBIOS HIPERTENSOS
EN EL FONDO DE OJO. Hay rales bibasilar sobre A) Diurético
el tercio inferior de los campos pulmonares; los
B) IECA
pulsos carotideos son normales. El impulso
apical es desplazado lateralmente y sostenido. C) Calcio antagonista (contraindicados en falla
S1 y S2 son normales. Hay un fuerte S4 y S3. cardiaca)
Existe un soplo holosistólico grado 2/6 que
D) Betabloqueador
irradia a la axila. El resto de los resultados del
examen son normales, excepto un leve edema E) ARA2
pedio. La radiografía de tórax muestra el
agrandamiento ventricular izquierdo (LV). El ECG
es consistente con la hipertrofia LV. La causa
Mujer de 72 años ingresa a emergencia por
más probable de la insuficiencia cardíaca de
dolor toráxico típico, de 6 horas de evolución,
este hombre es:
PA:130/70; FC:80x’; Sat O2: 97%; el
A) Hipertensión arterial electrocardiograma muestra supradesnivel del
segmento ST de 2mm de V1 a V4 con elevación
B) Miocardiopatía alcohólica enzimática de Troponinas; el tratamiento
inmediato es:
C) Ateroesclerosis coronaria
A) Anticoagulación oral
D) Miocardiopatía hipertrófica
B) Trombolisis farmacológica
E) Dosis excesiva de betabloqueante
C) Hipolipemiantes
D) Antagonistas receptores de angiotensina II C) Insuficiencia Válvula Mitral

E) Betabloqueantes D) Estenosis Válvula Mitral

E) Estenosis Válvula Aórtica

Paciente varón de 43 años, tabaquista,


diabético. Ingresa a emergencia por dolor
BANCO 2020-2
torácico opresivo, se realiza electrocardiograma,
se encuentra supradesnivel de segmento ST de Si un paciente muestra estabilidad
V3 a V6. Examen: PA 80/40, FC: 120x’, Sat O2: hemodinámica y en el EKG se observa TPSV se
89%, ¿Qué indicaría Ud.? debería:
A) Fibrinolisis farmacológica A) Intentar maniobras vagales
B) Inhibidores de Enzima Convertidora de B) Administrar adenosina
Angiotensina
C) Administrar verapamilo
C) Nitratos endovenosos
D) Administrar amiodarona
D) Intervencionismo Coronario Percutáneo
E) Administrar digoxina
E) Betabloqueantes

Paciente varón de 31 años que ingresa con


tiempo de enfermedad de 5 días caracterizado
por dolor toráxico tipo punzada de intensidad
5/10 que aumenta con la inspiración. Paciente
presenta 3 días antes del evento mencionado
cuadro de resfrío y gastroenterocolitis sin mayor
complicación. Funciones vitales: FC 98 x´FR 20
x´ Sat02 98 % a FIO2 21 %. Presenta un EKG
donde se observa en ritmo sinusal con elevación
del segmento ST en precordiales anteriores,
laterales e inferiores y un ST infra desnivelado
en aVR. ¿El diagnóstico más probable es?

A) Tromboembolia pulmonar
¿Cuál es la cardiopatía congénita de alto flujo
B) Sica ST no elevado con imagen de patrón de más frecuente del recién nacido?
tronco de coronaria izquierda
A) Tetralogía de Fallot
C) Pericarditis Aguda
B) Comunicación interauricular
D) Síndrome de Takotsubo
C) Transposición de grandes vasos
E) Osteocondritis asociada a patrón
D) Comunicación interventricular
electrocardiográfico de repolarización precoz
E) Persistencia del conducto arterioso
Paciente de 68 años de edad, con angina de
pecho, insuficiencia ventricular izquierda,
síncope, presión diferencial corta, soplo
eyectivo. Marque lo correcto: Niño de 18 meses de pobre ganancia de peso,
desde hace 3 meses madre se percata de color
A) Comunicación Interauricular azulado de su piel. Lo lleva a emergencia donde
al examen físico encuentran FC: 150 lpm, FR: 50
B) Persistencia del Ductus arterioso
rpm, Sat Oxígeno 80 %, cianosis. ¿Su
diagnóstico de cardiopatía congénita más C) El diagnostico etiológico del cuadro clínico
probable es? probablemente sea una cardiomiopatía

A) Transposición de grandes arterias D) El examen físico sugiere enfermedad valvular


regurgitante asociada a lesión estenótica
B) Tetralogía de Fallot
E) El primer ruido disminuido sugiere flexibilidad
C) Atresia tricuspídea
importante de los velos de la válvula
D) Coartación de aorta comprometida

E) Ventrículo único Mujer de 37 años con antecedentes de


Fibrilación atrial e hipertensión arterial bajo
tratamiento con losartan y warfarina. Ingresa a
emergencia por presentar fiebre, ortopnea y
Con respecto a las cardiomiopatías dilatadas,
dolor torácico atípico con evolución desde hace
señale lo correcto:
1 semana, presentando hoy sincope. Al ingreso a
A) Son debido a valvulopatías (insuficiencias shock trauma se constata PA 70/60, FC 132 lpm,
valvulares) FR 35 rpm y 85 kg de peso, SO2 85%, se le
realiza: ELECTROCARDIOGRAMA de urgencia
B) Presentan principalmente disfunción mostrando elevación del Segmento ST difusa
diastólica con complejos QRS pequeños.
C) La cardiomiopatía periparto siempre está ECOCARDIOGRAFÍA se evidencia una efusión
asociada a eclampsia pericárdica circunferencial con diámetro mayor
de 40 mm y colapso diastólico del Ventrículo
D) La cardiomiopatía de Tako Tsubo está derecho. LABORATORIO PCR 9, INR 2.5,
asociada a obstrucción coronaria (observada en creatinina 1.1 mg/dl, lactato 5 mg/dl, troponina I
el cateterismo cardiaco) negativas al ingreso. ¿Cuál sería su diagnóstico
más probable?
E) La fracción de eyección está disminuida
A) Pericarditis crónica

B) Infarto agudo de miocardio ST elevado


Paciente de 45 años cursa con sensación de
palpitaciones en forma episódica desde hace 3 C) Efusión pericárdica moderado de riesgo alto
meses, nota que inicia con disnea a moderados
esfuerzos desde hace 1 mes. Al examen físico PA: D) Taponamiento pericárdico
110/40 FC: 88 lpm Pulmones: MV pasa bien en E) Pericarditis sub aguda
ACP CV: Impulso ventricular izquierdo en 5to
espacio intercostal izquierdo fuera de la línea Paciente mujer de 28 años de edad con
medio clavicular. Impulso ventricular derecho antecedentes de Hipertensión Pulmonar, PA:
negativo. Ruidos cardiacos totalmente 100/70 mm Hg, FC:88 por min. Cansancio a
arrítmicos. Primer ruido disminuido, soplo moderados esfuerzos, tos seca ocasional. Soplo
sistólico en 5to EICI LMC II/VI irradiado a axila, diastólico cardiaco II/VI. ¿Cuál de los siguientes
soplo diastólico en foco mitral II/VI. Abdomen hallazgos se encuentra en la paciente?
blando depresible, no visceromegalia. No
edemas. Pulso deficitario. Marque la respuesta A) Congestión Pulmonar
correcta: B) Chasquido de apertura
A) Según lo descrito en el examen físico es C) Soplo de Austin Flint
probable que haya crecimiento ventricular
derecho D) Soplo continuo

B) El soplo continuo descrito sugiere presencia E) Soplo Graham Steel


de ventana aortopulmonar
Joven de 16 años, presenta un cuadro de fiebre y
taquicardia, que se acompaña de un eritema
rosado, transitorio y migratorio, que no es
pruriginoso ni doloroso y que desaparece con la A) Ecocardiograma bidimensional
presión, la placa de tórax muestra un aumento
del corazón. La auscultación revela unsoplo B) NT Pro-BNP
mesodiastólico de tono bajo, que con frecuencia C) BNP sérico
aparece después de un tercer ruido aumentado.
La analítica demuestra un aumento de la VSG y D) MR Pro ANP
la proteína C reactiva. Señalar la afirmación
E) Todas las anteriores
CORRECTA respecto a su enfermedad:
Paciente varón 63 años, acude a consultorio de
A) Suele producir acortamiento del intervalo PR
Cardiología para control. Dx. por Historia Clínica:
B) El aumento de la incidencia de la enfermedad Insuficiencia Cardiaca Crónica Compensada. Al
se asocia con mucha frecuencia al hacinamiento estudio Ecocardiográfico: FE 35%. Antecedente
de IMA previo, FA (-), HTA (+), intolerante a IECAs,
C) La válvula Aórtica es la que con más
estable hemodinámicamente, sin congestión
frecuencia se asocia
pulmonar y con función renal preservada; sin
D) La alteración del pericardio produce hiperkalemia, edema de MMII (-). ¿De las
constricción pericárdica siguientes combinaciones de fármacos, cuál
considera usted sería inicialmente la más
E) En la biopsia pulmonar son típicos los cuerpos indicada?
de Aschoff
A) Levosimendán + digitálicos

B) Antagonistas de la aldosterona + dopamina


Paciente varón de 60 años con antecedente de
Diabetes mellitus y enfermedad renal crónica C) Dobutamina + Antagonistas de los receptores
estadio 3 (DC: 50 ml/min/m2) es derivado a de la angiotensina (ARA II)
consultorio de medicina para evaluación
D) Betabloqueadores + Bloqueadores de los
integral. Al Examen: PA 150/90 mmHg. ¿Cuál será
Receptores de Angiotensina (BRAs)
la mejor medida a tomar?
E) Inhibidores ECA + Diuréticos de ASA
A) Régimen higiénico dietético y control de PA en
6 meses Paciente femenina 61 años, quien es ingresada
a Emergencia por presentar cuadrode
B) Dieta baja en sal, control de glicemia y control hemorragia digestiva aguda de 2 horas de
en 1 año evolución sin repercusión hemodinámica. Al
C) Tratamiento farmacológico de inicio y régimen realizarse la endoscopia digestiva no se
higiénico dietético encuentra al momento punto de sangrado activo.
Refiere la paciente posteriormente dolor
D) Control dietético y de sal, y luego de 3 meses precordial de inicio súbito acompañado de
iniciar fármacos disnea y ortopnea. Al EKG: ritmo sinusal con
presencia de supradesnivel del segmento ST en
E) Ninguna de las anteriores
V1, V2, V3, y V4. Radiografía de Tórax: marcada
En la evaluación inicial de un paciente con congestión pulmonar bilateral. Se procede a
síndrome de insuficiencia cardiaca es intubación orotraqueal e instalación de soporte
necesario determinar algunas pruebas básicas ventilatorio mecánico e indicación de diuréticos,
como electrolitos, pruebas hepáticas, indicándose tratamiento médico para IMA STE
Hemograma, EKG y radiografía de tórax. al no contarse con facilidades para realización
También existen asociadas a estas algunos de cinecoronariografía y posibilidad de
procedimientos y pruebas que se realizaran en angioplastia con balón. Paciente evoluciona con
una segunda etapa que son fundamentales para inestabilidad hemodinámica a pesar de recibir
llegar a un diagnostico etiológico definido y apoyo inotrópico y vasopresor. Se plantea Dx.
realizar un manejo optimo. Marque Ud. la Falla Cardíaca Aguda post IMA STE.¿Qué otra
respuesta CORRECTA: medida sugeriría usted para el tratamiento de
esta paciente?
A) Fibrinólisis E) Aspirina, Prasugrel, Warfarina

B) Balón de Contra pulsación Intra Aórtico


(BCIAo)
Paciente sometido a Intervencionismo
C) Cardio Desfibrilador Implantable Percutáneo Coronario (colocación de Stent en
arteria Descendente Anterior) por Infarto Agudo
D) Terapia de Resincronización Ventricular de Miocardio ST elevado.Marque la indicación
correcta:
E) Todas las anteriores
A) Heparina no fraccionada por un mes

B) Clopidogrel más Ticagrelor por una semana


De los tipos de shock cardiogénico, de acuerdo
con el status volémico y la circulación periférica, C) Warfarina más Aspirina por un año
hemos descrito al shock clásico con una
disminución del índice cardiaco, aumento de las D) Doble antiagregación plaquetaria por un año
resistencia vascular sistémica y aumento de la o más ⇒ el TAPD (AAS + Clopidogrel u otro
presión capilar pulmonar enclavada o presión inhibidor del P2Y) está indicado hasta 1 año para
de Wedge. Esto sería un shock: pacientes tratados con fibrinolisis y después de
ICP.
A) Frío-Húmedo
E) Doble antiagregación plaquetaria solo por 3
B) Frio-Seco semanas

C) Caliente-húmedo Paciente varón de 66 años, con antecedente de


HTA, ingresa a emergencia por dolor torácico
D) Caliente-Seco típico de pocas horas de evolución, se halla
E) Frio-Caliente estable hemodinámicamente, EKG sin alteración,
enzimas cardiacas discretamente elevadas, ¿su
El paciente anterior presenta PA: 80/40 MmHg, presunción diagnostica es?
FC: 90x’, Sat O2: 88%; con incremento del dolor
toráxico, solicita nuevo EKG:Cual es la conducta A) Tromboembolismo pulmonar
terapéutica indicada:
B) Síndrome isquémico agudo
A) Administro IECAs
C) Miocarditis
B) Administro nitratos
D) Infección por COVID-19
C) Intervencionismo coronario percutáneo
E) Todas las anteriores
D) Fibrinolisis farmacológica
Paciente varón de 60 años que acude a consulta
E) Todas las anteriores por opresión en el centro del pecho irradiado a
la cara anterior del cuello que se presenta luego
Paciente 60 años con 30 minutos de dolor de subir escaleras, refiere ser hipertenso y
toráxico, el EKG muestra elevación del segmento diabético en tratamiento con valsartan,
ST de 2mm en varias derivaciones. ¿Cuál es el metformina, empagliflozina y atorvastatina.
tratamiento indicado? ¿Cual considera la conducta a seguir?
A) Fibrinolítico o angioplastia percutánea, doble A) Agregar aspirina al tratamiento
antiagregación plaquetaria, beta bloqueantes
B) Tomar electrocardiograma
B) Heparina sódica, Acido acetil salicílico,
Inhibidor glucoproteína IIb/IIIa. C) Realizar una prueba de esfuerzo

C) Lidocaína EV, Oxigeno, Amiodarona D) Complementar la evaluación con un


ecocardiograma
D) Heparina de bajo peso molecular, doble
antiagregación plaquetaria, Warfarina E) Todas ellas
Varón de 65 años, obeso, a la mitad de un viaje Mayor riesgo quienes tuvieron un episodio
interprovincial de Tumbes a Tacna presenta anterior
disnea y dolor torácico, es llevado a un hospital ● Muerte súbita
de Lima, donde le encuentran PA: 80/50mmHg,
FC: 110x´, SatO2 87%, Radiografía de tórax ● Síncopes recurrentes
presenta algunas atelectasias laminares. Llama
● Arritmias ventriculares
la atención un edema marcado de MI izquierdo
con dolor en la pantorrilla. Teniendo en cuenta ● Historia familiar de muerte súbita
los datos clínicos y los signos de severidad que
● Hipotensión en el ejercicio
puedan estar presentes, diga usted cual es el
examen que solicitaría inicialmente para ● Septum mayor de 30 mm
determinar el diagnóstico del paciente:
2. El síndrome de insuficiencia cardiaca se
A) Angiotomografía pulmonar caracteriza por presencia de FEVI disminuida o
preservada, pero en clínica existen algunas
B) Electrocardiograma condiciones que generan falla cardiaca con
C) Gammagrafía pulmonar gasto cardiaco elevado. Marque lo correcto:

D) Dímero-D A) Anemia severa

E) Gases arteriales B) Sepsis

Paciente Varón de 35 años, presenta pulso C) Tirotoxicosis


saltón, danza arterial, disnea a pequeños
D) Enfermedad de Paget
esfuerzos, PA: 140/60 mm Hg. FC: 100 por min.
Soplo Diastólico cardiaco III/VI.EKG Crecimiento E) Todas son correctas
del Ventrículo Izq. Tele Rx. de Corazón y Grandes
Falla cardiaca asociada a GC alto:
vasos: Cardiomegalia. ¿Cuál es su Diagnóstico
más probable? ● Anemia severa
A) Estenosis Valvular Tricúspidea ● Cuadro séptico

B) Insuficiencia Valvular Aortica ● Tirotoxicosis

C) Insuficiencia Valvular Tricúspidea ● Fístula arteriovenosa

D) Persistencia del conducto arterioso ● Enfermedad de Paget

E) Insuficiencia Valvular Pulmonar ● Embarazo

3. En el tratamiento de la insuficiencia cardiaca


se indican los IECA y beta bloqueadores porque
EXAMEN DE CARDIOLOGÍA tienen efecto sobre la fisiopatología de la fibra
miocárdica insuficiente. Marque usted lo
1. No se considera criterio de riesgo de muerte
correcto:
súbita en personas con cardiomiopatías
hipertróficas: A) Disminuyen la hipertrofia de la fibra
miocárdica
A) Síncopes recurrentes
B) Disminuyen la necrosis miocárdica
B) Septum interventricular mayor de 30 mm
C) Disminuyen los mecanismos de apoptosis
C) Antecedentes familiares de muertes súbita
D) Desensibiliza a los receptores beta
D) Crisis hipertensiva durante el ejercicio
adrenérgicos
E) Arritmias ventriculares (taquicardia ventricular
E) Todas ellas
no sostenida)
4. Estamos en la calle y encontramos en el suelo
a un anciano de aproximadamente 80 años de
edad, luego de confirmar que está en parada
cardiorespiratoria, inicio a darle RCP básica con
compresiones torácicas. Marque lo correcto

A) Empezamos con compresiones suaves y con


baja frecuencia aprox. 50 por minuto

B) Comprimimos de 2 a 3 cm con mucho cuidado


de no romperle las costillas y a una frecuencia
de 60 por minuto

C) Comprimimos de 2 a 3 cm con mucho cuidado


de no romperle las costillas y a una frecuencia
de 100 por minuto

D) Comprimimos de 5 -6 cm, con riesgo incluso


de romperle las costillas y a una frecuencia de
100 a 120 por minuto
6. Paciente mujer de 62 años, ingresa a
E) Ninguna alternativa es correcta
emergencia por presentar dolor torácico de 3 hrs
de evolución, Troponinas elevadas,
electrocardiograma evidencia elevación del
5. Paciente varón de 44 años, con antecedente segmento ST en II, III, aVF.Tiene indicación de:
de tabaquismo, diabetes mellitus; ingresa a
emergencia por dolor toráxico opresivo. Se A) Trombolisis farmacológica
realiza electrocardiograma mostrando B) Angiotomografía de Coronarias
supradesnivel del ST de V3 a V6; al Examen
Físico: PA: 80/40; FC: 130 x’; Sat O2: 88% ¿Qué C) Ecocardiografía de stress
indicaría Ud?
D) Prueba de esfuerzo
A) Inhibidores de Enzima Convertidora de
E) Anticoagulación oral
Angiotensina

B) Nitratos
7. El síncope por hipotensión ortostática es
C) Intervencionismo Coronario Percutaneo
confirmado cuando en la prueba de
D) Betabloqueantes bipedestación activa se confirma:

E) Test de Perfusión Miocárdica A) Caída de la presión arterial sistólica de >20


mmHg o presión arterial diastólica de >10 mmHg
del valor basal

B) Hay una caída de la presión arterial sistólica


de >10 mmHg o presión arterial diastólica de >5
mmHg del valor basal

C) Hay una caída de la presión arterial sistólica


de >30 mmHg o presión arterial diastólica de
>20 mmHg del valor basal

D) Hay una caída de la presión arterial sistólica


de >50 mmHg o presión arterial diastólica de
>30 mmHg del valor basal

E) No hay caída de la presión arterial


8. En la clasificación de shock cardiogénico Señale cuál sería la decisión terapéutica inicial
según la sociedad americana de cardiólogos más importante:
intervencionistas (SCAI) el estadio “C”
A) Fondaparinux SC
corresponde a un shock:
B) Heparina sódica EV
A) En riesgo de desarrollar Shock, con síntomas
de infarto agudo de miocardio C) Warfarina VO
B) Clásico, caracterizado por hipoperfusión que D) Enoxaparina SC
requiere intervención (Inotrópicos, soporte
mecánico) además de volumen para restaurar la E) Alteplase ⇒ Usar terapia fibrinolítica en caso
hipoperfusión de TEP masivo (hipotensión, disfx VD, shock
cardiogénico)
C) Extremis con paro cardio-respiratorio
resucitado y sin respuesta 12. Recién nacido de 3 días de vida, cianótico,
con soplo sistólico, FC: 170 lpm, FR: 69 x´. Se da
D) Provocado por hipovolemia y pérdidas oxígeno suplementario y persiste cianótico. Su
importantes de sangre y/o volumen diagnóstico de cardiopatía congénita más
probable es:
E) Shock producido por una infección bacteriana
y/o viral severa A) Transposición de grandes arterias

9. Paciente varón de 68 años, conangina de B) Tetralogía de Fallot


pecho, insuficiencia ventricular izquierda,
síncope, presión diferencial corta, soplo C) Atresia tricuspídea
eyectivo. ¿El diagnóstico más probable es?
D) Coartación de aorta
A) Comunicación Interauricular
E) Ventrículo único
B) Persistencia del Ductus Arterioso
13. Las siguientes alternativas de tratamiento
C) Insuficiencia de válvula mitral son utilizadas en primera línea en el manejo de
síndrome isquémico coronario crónico, tras
D) Estenosis de válvula mitral haber recibido un stent de manera electiva,
EXCEPTO:
E) Estenosis de válvula aórtica
A) Ácido Acetil Salicílico
10. Varón de 68 años ingresa a emergencia por
presentar disnea de reposo, PA ; 145/50 mm Hg , B) Estatina
FC ; 100 lpm, Soplo diastólico , pulso saltón, EKG:
Hipertrofia Ventricular Izquierda,Tele Rx C y GV: C) Ticagrelor
Crecimiento de cámaras izquierdas. ¿Cuál es el D) Enoxaparina
diagnóstico más probable?
E) Enalapril (si coexiste insuficiencia cardiaca,
A) Insuficiencia valvular aórtica HTA o diabetes)
B) Estenosis valvular mitral Para prevención de eventos trombóticos:
C) Estenosis valvular tricúspidea ● Aspirina
D) Insuficiencia valvular Pulmonar ● Clopidogrel /Prasugrel/Ticagrelor

E) Insuficiencia valvular Tricúspidea ● Estatinas: estabiliza placa

11. Varón de 60 años, obeso y diabético, quién ● IECA, ARA 2: tto HTA, DM, IC
durante un viaje interprovincial presenta disnea
14. Paciente mujer de 27 años que ingresa con
súbita, llega a emergencia taquicárdico,
tiempo de enfermedad de 7 días caracterizado
taquipneico e hipotenso, la evaluación
por dolor torácico tipo punzada de intensidad
radiológica y laboratorial sugieren el
6/10 que aumenta con la inspiración y con la tos
diagnóstico de tromboembolismo pulmonar.
teniendo como antecedente de hace 5 días resistencia incrementada a la insulina. ¿Cuál
antes del evento mencionado cuadro de sería el mejor tratamiento?
gastroenterocolitis sin mayor complicación.
A) Dieta baja en sal, reducción de peso,
Niega alergias medicamentosas. Funciones
reducción del colesterol y control en 6 meses.
vitales: FC 118 x´, FR 20 x´, Sat02 98 % a FIO2
Aun no inicio tratamiento farmacológico
21%. Presenta un EKG donde se observa enritmo
sinusal con elevación del segmento ST en B) Cambio de estilo de vida, iniciar monoterapia
precordiales anteriores, laterales e inferiores y con ARA II en altas dosis y control en 6 semanas
un ST infra desnivelado en aVR.
Ecocardiograma: Función sistólica del VI C) Cambios de estilo de vida, iniciar terapia
conservada + motilidad conservada + Derrame combinada de ARA II y Antagonista del Calcio a
pericárdico leve. El Diagnóstico más probable y dosis estándar y control en 6 semanas
el tratamiento inicial de elección sería:
D) Cambios de estilo de vida, iniciar
A) Pericarditis aguda / AINES o ASA + corticoides monoterapia con diuréticos a altas dosis y
+ reposo físico control en 6 semanas

B) Sica ST no elevado con imagen de patrón de E) Cambios de estilo de vida, iniciar terapia
tronco de coronaria izquierda / combinada con metildopa y alfa bloqueadores a
Revascularización urgente dosis estándar con control en 6 semanas

C) Pericarditis Aguda / AINES o ASA + Colchicina 17. Paciente de 65 años con diagnóstico deICC
+ reposo físico ⇒ Tratamiento de primera linea por cardiopatía coronaria, tuvo un infarto que le
dejó una FEVI del 25%, está cursando con
D) Síndrome de Takotsubo / Manejo de Falla
Hipertensión arterial de 150/90. ¿Cuál será el
cardiaca
mejor tratamiento antihipertensivo?
E) Osteocondritis asociada a patrón
A) ARA II + Furosemida + Antagonista del calcio
electrocardiográfico de repolarización precoz /
Sintomáticos B) IECA + Furosemida + Antagonista de
aldosterona

C) Antagonista del calcio + Furosemida +


15. Varón de 12 años se presenta a consulta
Metildopa
ambulatoria por presentar fiebre de 38 °C,
monoartritis y eritema marginado en el dorso de D) ARA II + IECA + Antagonista de aldosterona
miembros superiores e inferiores. Como
antecedente refiere haber presentado hace 3 E) IECA + ARA II + B bloqueador
semanas un episodio de faringitis aguda, la cual 18. Paciente, varón de 70 años con antecedentes
fue tratada sólo con paracetamol. Respecto al de HTA en tratamiento con Losartán 50 mg/día,
diagnóstico y su complicación más frecuente a presenta disnea a leves esfuerzos.
largo plazo, señale la alternativa CORRECTA: Ecocardiograma: FEVI 30%. El tratamiento de
A) Faringitis – absceso faríngeo elección en este paciente sería:

B) Fiebre reumática aguda – valvulopatía aórtica A) IECA, calcio antagonistas, espironolactona

C) Fiebre reumática aguda – carditis B) ARA II, betabloqueantes, furosemida

D) Fiebre reumática aguda – cardiopatía C) IECA, betabloqueantes, hidroclorotiazida


reumática crónica D) IECA, betabloqueante, espironolactona
E) Enfermedad del tejido conectivo – artritis E) IECA, calcio antagonistas, hidroclorotiazida
crónica

16. Paciente varón de 40 años con HTA de


159/99 mmHg; obeso, dislipidémico, con
a) Desdoblamiento fijo del 1er ruido
cardiaco ( 2do ruido)

b) Su soplo sistólico es consecuencia


directa del CIA ( sistólico eyectivo por
hiperflujo pulmonar)

c) En el EKG su presentación más


frecuente es bloqueo completo de rama
izquierda ( bloqueo de rama derecha
incompleta)

d) Dilata cámaras izquierdas ( cámaras


derechas)

e) Flujo pulmonar elevado (hiperflujo


pulmonar)*

2. Una niña de 4 años que presentó infecciones


19. ¿Cuál de las siguientes alternativas, es
respiratorias frecuentes, el control de su peso y
criterio para terapia de resincronización
talla para su edad es bajo, presenta soplo en
cardíaca?
maquinaria de Gibson ¿Qué tipo de cardiopatía
A) Imagen de BRIHH congénita tiene esta niña?
B) QRS de amplitud mayor o igual 150 mseg a) Conducto del DA permeable
C) FEVI menor de 35 % (persistencia del conducto arterioso
PCA/PDA)*
D) Todas
b) CIA
E) Ninguna
c) CIV
Según la directriz, la única indicación clase I
para resincronizador es: Pacientes con FE ≤ 35% d) Coartación de aorta
Y ritmo sinusal Y morfología de bloqueo de rama
izquierdo Y QRS ≥ 150 ms Y síntomas a pesar de e) Anomalía de ebstein
la terapia optimizada.

20. En la emergencia ante un paciente con


cualquier tipo de Taquicardia lo primero que 3. El diagnóstico diferencial de la disección de
debemos evaluar es: la aorta se realiza con todos los siguientes:
A) Estado hemodinámico del paciente⇒ EXCEPTO
sabiendo esto podemos comenzar terapia
a. Sd coronario agudo
B) Si tiene QRS ancho
b. Regurgitación aórtica sin disección
C) Si tiene QRS angosto
c. Aneurisma aórtica sin disección
D) Si es rítmica
d. Pericarditis
E) Si es arrítmica
e. Neumotórax *
PREGUNTAS SÁBADO C:*?

1. Cuál es característico en un paciente con


4. Un paciente de 61 años presenta dolor
CIA:
anginoso típico de 35 minutos de duración
siendo atendido en emergencia del hospital con examen auxiliar indicaría en el paciente?
una PA de 110/60, pulso 86xmin, no tiene signos
de insuficiencia cardiaca, de todas las medidas a. Angioresonancia
que se van a señalar ¿Cuál usted no indicaría?
b. Flebografía
a. Canalizar vía periférica c. Eco doppler de venas **
b. Tratar el dolor con morfina d. Estudio del fibrinógeno
c. Ponerle un antiarrítmico parenteral e. Radiografía del miembro afectado
d. Administrarle oxígeno
9. El soplo de Graham Steel en que enfermedad
cardiaca se observa:(EM-
diastolico-insuficiencia tricúspide y pulmonar)
7. ¿Cuál de los medicamentos son de elección
para el control de la HTA enun paciente con a. Insuficiencia valvular aórtica
varios episodios de insuficiencia cardiaca
b. Insuficiencia valvular mitral
congestiva?
c. Atresia tricuspídea
a. Diltiazem (antiarritmicos-clase III)
d. Insuficiencia valvular pulmonar
b. Propranolol (B-Bloq-no selectivo)*
e. Estenosis valvular aórtica
c. Clortalidona (Diuretico-tiacidico)*

d. Hidroclorotiazida (diuretico- tiacidico)*


10. Signo de musset es característico de cuál de
e. Enalapril (IECA)**
los siguientes:
Como terapia inicial: ARA II o Bloqueadores de a. Estenosis valvular mitral
los receptores de la angiotensina II +
betabloqueadores + diuréticos (si está b. Insuficiencia venosa profunda
congestivo). c. Insuficiencia valvular aórtica

Pacientes que tienen baja fracción de eyección o d. Estenosis valvular tricuspídea


que hacen hipertensión resistente: Utilizar
e. Insuficiencia cardiaca derecha
Antagonistas de los mineralocorticoides osea
espironolactona. 11. Mujer de 58 años de edad con PA de 100/90,
FC 98 x min, paciente presenta cansancio a
moderados esfuerzos, tiene angina de pecho y
presenta DPN, y en el examen del corazón
encuentran un soplo sistólico cardiaco grado
III/VI. Señale el diagnóstico:

a. Insuficiencia valvular aórtica

b. Insuficiencia valvular tricuspídea

c. Estenosis valvular mitral

d. Insuficiencia valvular pulmonar

e. Estenosis valvular aórtica **


8. Paciente que llega a emergencia con un
cuadro de edema desde la raíz del muslo, en un
paciente que está postrado por un post
operatorio en el servicio de traumatología ¿Qué 12. Paciente que presente un soplo diastólico
con una presión diferencial amplia, un pulso de
corrigan o en martillo de agua. Señale lo para que pueda eliminar su volumen hacia el
correcto: ventrículo.

a. Estenosis valvular tricuspídea Pacientes sanos: 4-6 cm2


SM moderada (1 a 1.5 cm2) <2cm2:Presión
b. Estenosis valvular aórtica
auriculoventricular elevada
c. Insuficiencia valvular mitral SM grave (< 1cm2): Presión auricular izquierda
de 25 mmHg
d. Insuficiencia valvular aórtica

e. CIV
15. Recién nacido presenta una cardiopatía
congénita tendría lo siguiente:
CELER O EN MARTILLO DE AGUA: (patologías
regurgitantes) I. AÓRTICA, PCA, MAV a. Un soplo pansistólico
CONGÉNITAS, TIROTOXICOSIS, ANEMIA
b. Dilatación de cámaras derechas
13. Señale lo correcto:
c. Cianosis
a. El soplo de la insuficiencia valvular aórtica se
d. Presión arterial elevada en miembros
irradia al cuello
superiores y disminuidas en inferiores
b. El soplo en la estenosis valvular pulmonar se e. Pulso periférico amplio o céler **
da en el 4to espacio intercostal izquierdo
16. Cuál de los siguientes es un criterio mayor de
c. El soplo de la insuficiencia
una endocarditis infecciosa:
valvular pulmonar se irradia a
la axila a. Fiebre mayor de 38 °C
d. La presión diferencial corta
b. Solo 1 hemocultivo positivo
se da en la estenosis valvular
aórtica* c. Embolia arterial recidivante
e En la estenosis valvular d. Cardiopatía predisponente
pulmonar el soplo se da en la
fase diastólica e. Hallazgo de vegetaciones
intracardiacas por ecocardiograma
transtorácico **
14. Señale lo correcto en la estenosis valvular
mitral:

a. El soplo se produce en la fase sistólica (soplo 17. Cuál es la causa más frecuente de
diastolico)
muerte súbita cardiaca:
b. Hay sobrecarga de volumen en la aurícula
izquierda (de presión en la auricula izq) a. Miocardiopatía hipertrófica
c. Existe un chasquido de apertura **(0.06 a 0.12 b. Tromboembolismo pulmonar
secs)
c. Disección de aorta
d. Hay sobrecarga de volumen en el ventrículo
izquierdo (de presión auricula izq) d. Cardiopatía isquémica **

e. El área valvular mitral es < de 1 cm2 en una e. Endocarditis infecciosa


estenosis leve
-MHC: Causa principal de muerte súbita en
pacientes mayores de 35 años.
La aurícula izquierda ejerce una sobrecarga de
presión (produce hipertrofia auricular izquierda) 18. Señale la alternativa verdadera en relación a
síncope: los 52 años, este paciente presenta un colesterol
HDL de 31 mg/dL ¿Cuál de los siguientes valores
a) Hay flacidez completa elegiría usted como objetivo de tratamiento?
b) Es frecuente que el paciente en relación a. Colesterol total
a síncope resulte con mordedura de la
lengua. b. HDL

c) Es frecuente la aparición de c. LDL


incontinencia urinaria d. Triglicéridos
d) Hay recuperación inmediata de lucidez 22. Los betabloqueadores son útiles para tratar
posterior al evento ** la angina de pecho, esto debido a que:
19. Cuál de los siguientes criterios durante la a. Ser selectivos en prevenir el espasmo
evaluación del paciente con síncope considera coronario
usted que NO es de alto riesgo:
b. Su marcado efecto vasodilatador arterial
a. Historia familiar de muerte súbita coronario
b. Diagnóstico de insuficiencia cardiaca previo c. Aumentar el consumo de oxígeno
c. Edad de 15 años sin comorbilidad asociada** d. Disminuir la frecuencia
cardiaca y la
d. Infarto miocárdico en el último mes
contractilidad
e. Un electrocardiograma con una onda QT de miocárdica
0,49 seg
e. Incrementar la
20. Cuál de las siguientes entidades o frecuencia cardiaca
situaciones se asocian a un síncope
neuromediado:

a. Estenosis aórtica crítica

b. Bloqueo AV de tercer grado

c. Taquicardia ventricular no sostenida

d. Post micción

e. Enfermedad de Parkinson

23. El en algoritmo universal de RCP, cuál es la


secuencia correcta:

a. Dar 30 compresiones torácicas

b. Encender el desfibrilador en
cuanto pueda y seguir sus
instrucciones

c. Evaluar respuesta y
21. Paciente de 55 años con enfermedad respiración del paciente
coronaria crónica cuyo padre falleció de IAM a
d. Llamar al servicio de urgencia médica 112

e. Dar 2 respiraciones de rescate

f. Continuar con la RCP dar 30 a 2

24. En relación al tratamiento de la hipertensión


arterial:

a. es adecuado combinar una beta bloqueadora


con un calcioantagonista no dihidropiridínico

b. es adecuado combinar un IECA con un ARA II

c. es adecuado combinar un IECA con un calcio


antagonista

d. En HTA estadio II no se debe usar diuréticos

e. en personas de raza negra es conveniente


iniciar tratamientos con un IECA

26. En un paciente con angina de pecho estable


típica, lo más aconsejable es:

a. angiografía coronaria inmediata para


confirmar sospecha

b. dar tratamiento médico y evaluar en 6 meses


25. Mujer de 60 años diabética , hospitalizada
por celulitis , quien presenta disnea durante su c. electrocardiograma
internamiento , le aplican la escala de Wells para d. solicitar marcadores de injuria miocárdica
evaluar la posibilidad de tromboembolia
pulmonar dando un puntaje de baja e. estratificar riesgo con electrocardiograma de
probabilidad ¿Cuál sería el examen a solicitar esfuerzo
para el descarte de esta patología ?
27. En uno se los siguientes tipos de bloqueo AV
a. radiografia de torax existe alargamiento progresivo del PR

b. gammagrafía pulmonar a. primer grado

c. dimero D b. mobitz I fenómeno de Wenckebach (BAV 2do


grado mobitz I)
d. angiotomografía pulmonar
c. mobitz II
e. arteriografía pulmonar
d. bloqueo de alto grado

e. tercer grado

28. Varón de 25 años que presenta bruscamente


taquicardia de 200 lpm con severa hipotensión.
En el EKG se encuentra taquiarritmias QRS
ancho y de ascenso irregular ¿Cuál es el
tratamiento inicial recomendado? Parece ser
una Fibrilación Ventricular, hemodinámicamente
inestable. b. electrocardiograma el cuadro parece ser de
pericarditis aguda (dolor punzante) por lo que su
a. isosorbide EKG podría mostrar ST elevado cóncavo, y
serviría para diferenciarlo de un infarto con
b. digoxina
dolor atípico con elevación ST convexa.
c. nitroglicerina
c. ecocardiograma
d. verapamilo
d. prueba de esfuerzo
e. cardioversión asincrónica (desfibrilación) a
e. prueba de holter
300-400 joules con RCP.

31) En relación a la auscultación cardiaca,


29. Mujer de 73 años , con antecedente de HTA marque lo incorrecto:
portador de prótesis de válvula mitral biológica.
tiempo de enfermedad 1 mes caracterizado por a) El tercer ruido cardiaco puede ser
disnea , dolor torácico y palpitaciones , examen normal en niños.
clínico : PA 120/80 FC : 115 lpm , edema de
miembros inferiores , tórax: crépitos bibasales, b) El hallazgo de thrill permite clasificar la
cardiovascular: ruidos cardiacos arrítmicos . 1er intensidad de los soplos.
ruido cambiante , EKG : fibrilación auricular con
respuesta ventricular variable , FE : menor 45% , c) El tercer ruido cardiaco ocurre por
el manejo más adecuado es: alteración importante a nivel de las
aurículas.
a. antiagregante , diuréticos , beta bloqueador ,
diurético d) En la doble lesión aórtica, el soplo
b. IECA , anticoagulación , digoxina , diurético diastólico en foco aórtico es
Para Fib. Aur.: Digoxina manifestación de la insuficiencia
valvular.
Para prevenir trombos por Fib. Aur.:
Anticoagulantes (NO ANTIAGREGANTES, no son e) La auscultación de soplo continuo en
reemplazables) foco pulmonar irradiado a la espalda
sugiere la posibilidad de persistencia
Para los signos y síntomas de congestión:
de un conducto arterioso
Diuréticos

Supongo que IECAs para evitar que la PA (ANSWER: C )


aumente sobre el rango normal.

c. IECA ,anticoagulación, ivabradina , diurético

d. digoxina , diurético , antiagregante , IECA

e. IECA, digoxina, antiagregante ,


calcioantagonista

30. Varón de 48 años con antecedentes de


tabaquismo y diagnóstico reciente de
hipertensión arterial . consulta por dolor tipo
punzada a nivel precordial . al examenclínico no
demuestra hallazgos de significación .corazon
tonos normales , con ritmo regular ¿Cuál es el
examen más recomendado para valoración
diagnóstica?

a. estudio de medicina nuclear


2022-1 A) Observar a la enferma en una habitación
tranquila
Paciente con Dx Taquicardia Paroxística
Supraventricular, estable hemodinámicamente, B) Esperar los resultados de laboratorio antes de
fallan las maniobras vagales, la siguiente decidir el tratamiento especifico
medida es:
C) Administrar Nitroglicerina en infusión EV
A) Adenosina
D) Administrar diazoxido sódico en bolo EV
B) Cardioversión eléctrica
E) Administrar sulfato de Magnesio EV
C) Digoxina
ANSWER: C
D) Betabloqueantes

E) Calcio antagonistas dihidropiridinicos


Varón de 54 años, hace un mes desarrolló tos no
productiva después de que el paciente realizó
un viaje de negocios a Huancayo. Durante la
Paciente varón de 40 años con HTA de 159/99
semana siguiente, la tos se volvió productiva de
mmHg, obeso, dislipidémico, con resistencia
esputo intermitente de color marrón amarillento
incrementada a la insulina. ¿Cuál sería el mejor
y se asoció con fatiga y disnea. El paciente
tratamiento?
pensó que tenía bronquitis y dejó de fumar.
A) Dieta baja en sal, reducción de peso, Durante las siguientes 3 semanas, tuvo fatiga
reducción del colesterol y control en 6 meses. debilitante progresiva, anorexia, 10 kg de
Aun no inicio tratamiento farmacológico aumento de peso, dificultad para dormir
mientras estaba acostado y edema en ambas
B) Cambio de estilo de vida, iniciar monoterapia piernas. La disnea del paciente empeoró, fue
con ARA II en altas dosis y control en 6 semanas provocada ante cualquier esfuerzo y luego
C) Cambios de estilo de vida, iniciar terapia comenzó a ocurrir en reposo. Se había
combinada de ARA II y Antagonista del Calcio a despertado varias veces con disnea súbita.
dosis estándar y control en 6 semanas Temperatura de 36,6 °C, frecuencia cardíaca de
107/minuto, presión arterial de 155/73 mm Hg,
D) Cambios de estilo de vida, iniciar frecuencia respiratoria de 28/minuto y
monoterapia con diuréticos a altas dosis y saturación de oxígeno del 96% mientras recibía
control en 6 semanas oxígeno a través de una cánula nasal 2 litros/
minuto. al examen crepitantes en ambos campos
E) Cambios de estilo de vida, iniciar terapia
pulmonares y edema en las piernas. Estatura
combinada con metildopa y alfa bloqueadores a
normal, ansioso, fatigado, diaforético y dificultad
dosis estándar con control en 6 semanas
respiratoria leve. El pulso venoso yugular se
elevó a 12 cm de agua, y el pulso carotideo se
describió como saltón. El punto de impulso
Paciente Mujer de 68 años, es traída a cardíaco máximo se desplazó lateralmente a la
emergencia por que hace 24 horas se encuentra línea axilar anterior. Frecuencia cardíaca
Incoherente, con historia antigua de diabetes taquicardia; soplo en decrescendo (grado 2 de
mellitus 2 e hipertensión arterial, la familia 4) se escuchó casi en toda la diástole y fue más
refiere que ha dejado de tomar los prominente en el borde esternal superior
antihipertensivos. A la exploración se observa derecho, y no hubo galope S3. Crepitantes
una paciente desorientada con PA: 230/160 pulmonares bibasales. Los brazos y las piernas
mmHg. FR 26 rpm, FC 110 lpm y T: 36.7°C. A la tenían pulsos robustos y simétricos y eran
auscultación de pulmones hay crepitantes cálidos; hubo edema simétrico de 3+ en piernas.
bibasales. Se monitoriza a la paciente y se El examen de las uñas reveló una palidez alterna
insertan vías endovenosa y arterial. Una TEM de enrojecimiento y blanqueamiento que se
cerebral excluye hemorragias y masa sincronizó con la frecuencia cardíaca. No hubo
intracerebral. ¿Cuál de las siguientes, es la erupción cutánea ni linfadenopatía. Resto del
Indicación más adecuada?
examen fue normal. El diagnostico probable es: ANSWER: C

A) Insuficiencia cardiaca secundaria a estenosis


mitral
El síncope por hipotensión ortostática es
B) Insuficiencia cardiaca secundaria a confirmado cuando en la prueba de
insuficiencia mitral bipedestación activa se confirma:

C) Insuficiencia cardiaca secundaria a A) Caída de la presión arterial sistólica de > 20


insuficiencia aórtica mmHg o presión arterial diastólica de > 10
mmHg del valor basal
D) Insuficiencia cardiaca secundaria a estenosis
aortica B) Hay una caída de la presión arterial sistólica
de > 10 mmHg o presión arterial diastólica de > 5
E) Insuficiencia cardiaca secundaria a
mmhg del valor basal
cardiomiopatía dilatada
C) Hay una caída de la presión arterial sistólica
ANSWER: C
de > 30 mmHg o presión arterial diastólica de >
20 mmhg del valor basal

Una de las pruebas con más alta sensibilidad y D) Hay una caída de la presión arterial sistólica
especificidad que permite descartar o de > 50 mmHg o presión arterial diastólica de >
diagnosticar un cuadro clínico de insuficiencia 30 mmHg del valor basal
cardiaca aguda es:
E) No hay caída de la presión arterial
A) Troponina t
ANSWER: A
B) Proteína C reactiva

C) Bnp
Paciente varón 62 años, quien luego de ser
D) Nt-probnp trombolizado con activador del plasminógeno
tisular (TPA) por presentar cardiopatía isquémica
E) Mr-anp aguda con ST elevado, presenta súbitamente
dolor torácico e hipotensión arterial. La
ANSWER: D
indicación por seguir sería:

A) Intervención Coronaria Percutánea


Usted se encuentra de guardia en la unidad de
B) Ecocardiografía de stress
cuidados intensivos coronarios y la enfermera le
avisa que un paciente ingresado por IMASTE, C) Prueba de esfuerzo
con edema agudo de pulmón, intubado,
súbitamente presenta hipotensión. Usted acude D) Trombólisis farmacológica
a evaluarlo y en el monitor observa un trazado
E) Perfusión Miocárdica
compatible con fibrilación ventricular. Marque la
alternativa correcta. ANSWER: A

A) Usted indica administrar un reto de Cloruro de


Sodio 9 0/00, 300 ml EV STAT
Paciente mujer de 65 años, ingresa a
B) Usted indica colocar al paciente en posición emergencia por presentar disnea en reposo y
de Fowler dolor torácico opresivo 7/10 de intensidad, al
momento de la evaluación la paciente presenta
C) Usted procede a desfibrilar al paciente
fibrilación ventricular por lo que requirió de
D) Todas las anteriores desfibrilación eléctrica saliendo en sinusal con
hipotensión y desaturación, el
E) Ninguna de las anteriores electrocardiograma presenta infradesnivel del
segmento ST de V1 a V6. Ud indicaría:
A) Trombolisis farmacológica cardiopatía isquémica, es llevado a emergencia.
Se encuentra con alteración del estado de
B) IECAs
conciencia: PA 80/50 mmHg. Al monitor:
C) Nitroglicerina endovenosa Taquicardia QRS ancho. La acción inmediata es:

D) Cateterismo cardiaco A) Administrar Amiodarona

E) Betabloqueantes B) Administrar Adrenalina

ANSWER: D C) Cardioversión eléctrica

D) Maniobras vagales

Mujer de 68 años ingresa por emergencia E) Fluidoterapia


presentando disnea al reposo. PA: 140/50
ANSWER: C
mmHg. FC: 100 lpm, soplo cardiaco en fase
diastólica, pulso celer, EKG; Hipertrofia
Ventricular izquierda. Tele Rx C y GV: Crecimiento
Paciente de 25 años quien es usuario de drogas
de cámaras izquierdas. Indique el diagnóstico
ilícitas endovenosas, se le diagnostica
más probable:
endocarditis infecciosa. ¿Cuál es el
A) Insuficiencia valvular aortica microorganismo aislado con mayor frecuencia
en endocarditis infecciosa de pacientes
B) Estenosis valvular mitral
drogadictos intravenosos y cual la válvula
C) Estenosis valvular tricuspídea afectada más frecuentemente en estos
pacientes?
D) Insuficiencia valvular Pulmonar
A) Estafilococos aureus - Válvula mitral
E) Insuficiencia valvular tricuspídea
B) Streptococo viridans - Válvula mitral
ANSWER: A
C) Estafilococos epidermidis - Válvula aórtica

D) Cándida - Válvula tricuspídea


Mujer que luego de presenciar un accidente de
tránsito presenta dolor retroesternal opresivo de E) Estafilococo aureus – Válvula tricuspídea
severa intensidad por lo que acude por
ANSWER: E
emergencia. En el EKG se evidencia
supradesnivel del ST en derivadas precordiales
y en el ecocardiograma se ve aquinesia apical
Mujer de 33 años, puérpera que luego de parto
(balonamiento apical). Señalar lo correcto:
vaginal presenta disnea progresiva hasta llegar
A) La arteria coronaria descendente anterior a la ortopnea. Al examen físico: PA: 80/90 mmHg.
debe tener obstrucción significativa FC: 112 lat/min. Pulmones: crépitos difusos en
ambos campos pulmonares. Se realiza
B) Probablemente el cuadro revierta en las
ecocardiograma donde se encuentra FE del VI:
siguientes semanas
35% y dilatación de cavidades cardiacas.
C) Las enzimas cardiacas serán normales Señalar lo correcto:

D) El pronóstico es malo (alta mortalidad) A) Se trata de un caso de eclampsia

E) Se trata de una cardiomiopatía por miocardio B) Lo más probable es que el cuadro no revierta
no compactada
C) Lo más probable es que haya tenido un
ANSWER: B cuadro de infarto del miocardio agudo

D) Como el cuadro es reversible no debo tratar la


insuficiencia cardiaca
Paciente de 60 años, con antecedentes de
E) Se debe aconsejar que la paciente no vuelva a VI conservada+ Derrame pericárdico moderado
tener otra gestación con liquido pericárdico trasudado. El
diagnóstico más probable y el tratamiento inicial
ANSWER: E
de elección sería:

A) Pericarditis crónica - AINES O ASA +


En un paciente de 52 años con diabetes, corticoides + reposo físico
hipertensión arterial, dolor de pecho al caminar
B) Sica ST elevado - Revascularización urgente
unas 10 cuadras que calma con el reposo. ¿Cuál
debe ser su primera acción por realizar de las C) Pericarditis Aguda - AINES O ASA + Colchicina
siguientes? + Reposo físico

A) Tomar un electrocardiograma D) Síndrome de Takotsubo - Manejo avanzado de


falla cardiaca
B) Solicitarle una radiografía de tórax
E) Osteocondritis asociada a patrón
C) Derivarlo a emergencia
electrocardiográfico de repolarización precoz -
D) Solicitarle TGO Y DHL Sintomáticos

E) Hacerle una tomografía por emisión de ANSWER: C


positrones

ANSWER: A
Paciente varón de 55 años que ingresa al
servicio de emergencia en mal estado general y
edematoso. Examen físico: PA de 70/50 FC:
Recién nacido de 3 días de vida, cianótico, con 127x´, FR 24x´, SAT02 96% FIO2 21%. Fascie
soplo sistólico. FC: 170 lpm, FR: 69X. Se da pálida. CV: RC taquicárdicos alejados de menor
oxígeno suplementario y persiste cianótico. Su intensidad, no se ausculta soplos, no galope. Se
diagnóstico de cardiopatía congénita más evidencia ingurgitación yugular importante. TyP:
probable es: MV pasa bien en ambos hemitórax. No ruidos
agregados. EKG: Taquicardia sinusal con signos
A) Transposición de grandes arterias
de alternancia eléctrica y derivaciones con poca
B) Tetralogía de Fallot amplitud. Se realizó ecocardiografía: separación
de hojas de pericardio de 30 mm con colapso de
C) Atresia tricuspídea cavidades derechas. El diagnóstico más
D) Coartación de aorta probable y el tratamiento inicial de elección
seria para este caso:
E) Ventrículo único
A) Derrame pericárdico leve - Conducta
ANSWER: A expectante

B) Derrame pericárdico moderado - Hidratación


para mejorar la PA
Paciente mujer de 22 años que ingresa con
tiempo de enfermedad de 8 días caracterizado C) Shock cardiogénico por disfunción
por dolor torácico tipo punzada de intensidad biventricular - Inotrópicos y vasoactivos e
6/10 que aumenta con la inspiración y con la tos ingreso a unidad de cuidados críticos
teniendo como antecedente de hace 5 días
antes del evento mencionado cuadro de catarro D) Shock obstructivo por taponamiento cardiaco
sin mayor complicación. Niega alergias - Pericardiocentesis evacuatoria inmediata con
medicamentosas. Funciones vitales: FC 124x´, FR estudio de líquido pericárdico e ingreso a
20x´m Sat02 98% a FIO2 21%. Paciente febril unidad de cuidados críticos
Presenta un EKG donde se observa en ritmo E) Pericarditis Constrictiva - Pericardiectomía
sinusal con presencia de signo de Spodick programada
positivo. Ecocardiograma: Función sistólica del
ANSWER: D

MARQUE LA RELACIÓN CORRECTA:

A) Pulso céler - Estenosis aórtica moderada

B) Pulso anácroto - Disfunción miocárdica grave

C) Pulso parvus tardus - Insuficiencia aórtica


severa

D) Pulso bisferiens - Fibrilación auricular

E) Pulso en martillo de agua - Insuficiencia


aórtica severa

ANSWER: E

Varón de 12 años se presenta a consulta


ambulatoria por presentar fiebre de 38° C,
monoartritis y eritema marginado en el dorso de
miembros superiores e inferiores. Como
antecedente refiere haber presentado hace 3
semanas un episodio de faringitis aguda, la cual
fue tratada sólo con paracetamol. Respecto al
diagnóstico y su complicación más frecuente a
largo plazo, señale la alternativa correcta:

A) Faringitis - Absceso faríngeo

B) Fiebre reumática aguda - Valvulopatía aórtica

C) Fiebre reumática aguda - Carditis

D) Fiebre reumática aguda - Cardiopatía


reumática crónica

E) Enfermedad del tejido conectivo - Artritis


crónica

ANSWER: D
PRÁCTICO En el tratamiento de la HTA, son indicaciones
especiales de los calcioantagonistas no
PASITO Nº1 HNHU
dihidropiridínicos todas los siguientes, excepto:
¿Cuál de los siguientes NO se considera un
a) HTA en pèrsonas de raza negra
factor de riesgo coronario?
b) Hipertrofia de ventrículo izquierdo
a) Alcohol
c) Control de las frecuencia ventricular en
b) Hipertensión arterial
la fibrilación auricular
c) Edad
d) Bloqueo AV de II/III grado
d) Tabaco
e) Hipertensión sistólica aislada en
e) Sexo personas de edad avanzada.

Usted examina un paciente traído por EMG y al En el ECG, el intervalo PR corresponde al


tomarle la presión arterial se le encuentra comprendido entre:

una disminución de la PAS durante la a) El final de la onda P y el inicio del QRS y


inspiración por lo que piensa en pulso su duración normal va de 0.12 a 0.20
paradójico. De las siguientes situaciones seg
clínicas, en cual NO se encuentra presente el
b) El inicio de la onda P y el inicio del QRS y
pulso paradójico?
su duración normal de 0.12 a 0.20 seg
a) Tromboembolismo pulmonar
c) El final de la onda P y el inicio de la R y
b) Pericarditis constrictiva su duración normal de 0.12 a 0.20 seg

c) Taponamiento cardiaco d) El inicio de la onda P y el final del QRS y


su duración normal es menor de 0.20
d) Eritema pulmonar (cor pulmonale)
seg
e) Estenosis aórtica ⇒ Pulso parvus et
e) El inicio de la onda P y el inicio del QRS y
tardus
su duración normal es menor de 0.12
Marque lo correcto en relación al ciclo cardiaco seg

a) El segundo ruido cardiaco es producido Marque lo correcto respecto a la hipertensión


por el cierre de las válvulas atrio arterial en base a las guías de la ISH del 2020
ventriculares 2º ruido semilunares
a) En < de 65 años la PA objetivos es < de
(mitral, bicúspide)
130/80 mmHg si lo tolera pero no menos
b) Durante la fase sistólica las presiones de 120/70 mmHg
en aorta son mayores a las del
b) Los calcioantagonistas están indicados
ventrículo izquierdo
en todos los casos de HTA con falla
c) La fase diastólica ocurre antes de la cardiaca con fracción de eyección baja
onda p del electrocardiograma
c) Se debe usar betabloqueadores en
d) El tercer ruido cardíaco se halla en los todos los pacientes con HTA y
pacientes jóvenes en niños en normal, enfermedad pulmonar obstructiva
adultos jóvenes crónica

e) Durante la diástole permanecen d) Considerar tratamiento con terapia dual


abiertas las válvulas atrioventriculares en pacientes de bajo riesgo con HTA
grado I menores de 50 años
Con respecto a la hipertensión arterial, marcar la
alternativa correcta:

a) La causa más frecuente de hipertensión


arterial es la secundaria

b) Los inhibidores de la
monoaminooxidasa y alimentos con
tiramina. No son causa secundaria de
hipertensión secundaria (inducida por
fármacos)

https://scc.org.co/wp-content/uploads/2020/06/ c) La causa menos frecuente de


GUIAS-ISH-DE-HTA-BOLETIN-144-.pdf hipertensión arterial son las
enfermedades recientes

d) La eritropoyetina es una causa


El electrodo que corresponde a la derivación V4
secundaria de hipertensión arterial
se coloca en:
inducida por fármacos ⇒
a) Línea axilar anterior a nivel del 5º Antiinflamatorios no esteroideos,
espacio intercostal izquierdo. Ciclosporina., Corticoides., Cocaína,
Anfetaminas, sustancias
b) Línea media clavicular a nivel del 5º simpaticomiméticas, Inhibidores de la
espacio intercostal izquierdo monoaminooxidasa y alimentos con
tiramina, Eritropoyetina, Alcohol
c) Línea paraesternal derecha a nivel del
4º espacio intercostal e) Los síndrome de sección medular no
son causas neurológicas secundarias
d) Línea paraesternal izquierda a nivel del
de hipertensión arterial
4º espacio intercostal
Respecto al tratamiento de la hipertensión
e) Línea axilar media a nivel del 5º esàcio
arterial, marque el enunciado correcto
intercostal izquierdo
a) Los calcioantagonistas no
dihidropiridínicos son las drogas de
elección ante un paciente hipertenso
con falla cardiaca con fracción de
eyección baja

b) Los IECAS están indicadas en pacientes


con hipertensión y estenosis de arteria
renal bilateral
c) Se recomienda utilizar IECAS asociadas izquierda de la desembocadura dela
a ARA 2 cuando hay hipertensión no vena cava inferior
controlada ya que se sinergizan sin
d) En la unión auriculoventricular el
efectos adversos importante
estímulo cardiaco experimenta un
d) El uso de betabloqueadores está retardo fisiológico de 0.06-0.10 seg que
contraindicado en paciente hipertenso permite un mayor llenado ventricular
con claudicación intermitente o bloqueo antes de su sístole
AV
e) Ante una despolarización del NSA
primero se activa la aurícula derecha
luego la aurícula izquierda y finalmente
el septum interauricular

COMENTARIO: A lo largo de tres haces


internodales (anterior o de Bachmann, medio o
de Wenckebach, y posterior o de Thorel), que son
miocardiocitos orientados longitudinalmente (no
e) Los IECAS son las drogas de elección en células del sistema específico de conducción), el
el tratamiento de hipertensión durante impulso sinusal llega rápidamente al nodo
el embarazo auriculoventricular, donde se produce un retraso
en la conducción del estímulo (segmento PR del
electrocardiograma)

Señale la opción que no corresponde al ritmo


sinusal normal en un ECG

a) Onda P positiva en DI, DIII y aVF y


negativo en aVR

b) Onda P precede a todos los QRS

c) Onda P de morfología normal

En relación al sistema de conducción cardiaca, d) Frecuencia cardiaca menor de 60 lpm


marque la afirmación correcta: con frecuencias auriculares y
ventriculares irregulares
a) El nodo sinusal tiene la forma de una
romboidea de 100-400 mm de largo y e) Frecuencia cardiaca entre 60-100 lpm
500 mm de espesor con frecuencias auriculares y
ventriculares regulares
b) La velocidad de conducción en el Haz
de Hiss es de 10-20 mm/seg en el
sistema de Purkinje 3000-4000 mm/seg En relación a electrofisiología cardiaca, al
y en el miocardio ventricular 3000-5000 periodo durante el cual solo un estímulo
mm/seg
eléctrico muy intenso puede producir respuesta
c) El nodo sinusal (NSA) en el marcapasos se denomina
cardíaco por su menor frecuencia de
despolarización espontánea; está a) Periodo refractario relativo
localizado en la unión del subepicardio
b) Periodo total de excitabilidad
de la aurícula derecha con la pared
c) Periodo refractario absoluto (Ningún afirmación FALSA:
otro estímulo es capaz de generar otra
a) Es útil para disminuir el tamaño de la
contracción por el miocardiocito)
zona infartada.
d) Periodo de no refractariedad
b) Es útil para limitar la disfunción
e) Periodo de excitabilidad supranormal ventricular izquierda.

c) Es útil para disminuir las arritmias.

PASITO Nº2 HNHU d) Es útil para disminuir la mortalidad.


Un paciente de 50 años con antecedentes de e) Sólo es útil en las primeras horas
tabaquismo, dislipidemia y diabetes, postinfarto.
consulta por presentar desde hace 36 horas En relación al score de GRACE en SICA sin
episodios repetidos de dolor torácico opresivo elevación del ST, marque la alternativa
con sudoración. El último y más prolongado tuvo
una duración de 45 minutos. El ECG muestra correcta:
descenso persistente del ST de 2 mm en
a) El score no permite calcular el riesgo de
derivaciones DII DIII y aVF.Señale la afirmación
muerte a los 6 meses según la
más correcta: el tratamiento inicial debe incluir
puntuación total al alta
(salvo contraindicación):
b) Un factor considerado en el score es la
a) Nifedipino, aspirina y clopidogrel.
presencia de paro cardíaco al ingreso
b) Únicamente aspirina.
c) Permite calcular el riesgo de muerte
c) Fibrinolisis, betabloqueantes y aspirina. intrahospitalaria en pacientes
(NO INDICADOS FIBRINOLÍTICOS) sometidos a coronariografía previa a la
hospitalización
d) Aspirina, clopidogrel y heparina.
d) Para su cálculo no se consideran como
e) Aspirina, clopidogrel, heparina y factores la creatinina sérica ni la
betabloqueantes. frecuencia cardíaca en reposo
Una mujer de 72 años acude a urgencias con un e) No se considera en la evaluación al alta
dolor torácico sugestivo de isquemia la presencia de infarto de miocardio
miocárdica de 4 horas de evolución. En el ECG se antigüo
observa un descenso del segmento ST de 2 mm
en V2-V6. ¿Cuál de las siguientes opciones
terapéuticas NO es adecuada?

a) Activador tisular del plasminógeno


(t-PA). (FIBRINOLÍTICO) ¡¡CUIDADO!!

b) Ácido acetilsalicílico.

c) Enoxaparina.

d) Clopidogrel.
¿Cuál de los siguientes factores no está
e) Heparina sódica.
asociado con un riesgo adicional de resultados
En relación con el empleo de la trombolisis en el adversos en un paciente diagnosticado con
infarto agudo de miocardio, señale la SICA SIN ELEVACIÓN DEL ST?
a) Haber tomado una aspirina todos los c) Ondas R altas en V1 - V3, elevación del
días durante los 3 meses previos ST en V7-V9 : infarto de cara anterior del
ventrículo izquierdo
b) Tener infradesnivel del segmento ST de
0,5mm en derivaciones V5 y v6 d) DI, aVL, V5-V6: infarto de cara lateral del
ventrículo izquierdo, ápex
c) Presentación a los 70 años
e) Derivación V1 y V2: infarto de cara
d) Tener antecedente de neumonía previa
lateral del ventrículo izquierdo
e) Tener antecedentes de hipertensión,
Son causas de supradesnivel del segmento ST
colesterol alto y diabetes
en el EKG:(marque la alternativa correcta)
Con cuál de las siguientes características puede
a) Injuria subepicárdica, hipotermia,
presentarse típicamente un infarto de ventrículo
hiperkalemia, pericarditis aguda.
derecho importante?
b) Hiperkalemia, miocardiopatías, injuria
a) Presión venosa pulmonar baja,
subendocárdica, pericarditis
estertores hasta mitad del tórax,
epistenocárdica.
hipotensión, elevación de ST en II, III y
aVF c) Hipokalemia, intoxicación digitálica,
marcapasos migratorio, fenómeno de la
b) Presión venosa pulmonar elevada,
concertina
auscultación pulmonar normal,
hipotensión, elevación del ST en d) Marcapaso, miocardiopatías,
derivación D III hipokalemia, sobredosis digitálica,
injuria subendocárdica.
c) Presión venosa pulmonar elevada,
auscultación pulmonar normal, e) Pericarditis aguda, hipotermia, injuria
hipertensión, elevación del ST en subepicárdica, hipokalemia
derivación D III hiperkaliúrica

d) Presión venosa pulmonar baja, ¿Cuál de los siguientes factores descalificaría a


auscultación pulmonar normal, un paciente con infarto de miocardio con ST
hipotensión, elevación del ST en Elevado de ser elegible para terapia fibrinolítica
derivación aVF en un hospital en el que se realiza PCI?

e) Presión venosa pulmonar normal, a) Menstruación activa


auscultación pulmonar normal,
b) Presentarse dentro de las 2 horas
hipotensión, elevación del ST en
posteriores al inicio de los síntomas
derivación D II
c) Falta de recursos para realizar una
Con respecto a los cambios típicos en el EKG en
Intervención coronaria percutánea (PCI)
el infarto de miocardio y la localización
dentro de los 90 minutos del primer
probable del infarto, marque la afirmación contacto médico
correcta:
d) Historia de malformación arteriovenosa
a) V5-V6 : infarto de septum intracraneal de 1 cm diagnosticada
interventricular hace 9 meses

b) D I, D III y avR : infarto de cara inferior e) Presentarse dentro de las 10 horas


del ventrículo izquierdo posteriores al inicio de los síntomas
Una mujer de 72 años se presenta con disnea, c) El nodo sinusal tiene la forma de una
presión arterial de 95/50 mm Hg, romboidea de 100 - 400 mm de largo y
500 mm de espesor.
elevación del segmento ST en V2 hasta V5 y
edema pulmonar bilateral en la radiografía de d) Ante una despolarización del NSA,
tórax. Presenta mayor hipotensión y luego primero se activa la aurícula derecha,
obnubilación requiriendo intubación. Se le luego la aurícula izquierda y finalmente
prepara para PCI emergente. Un el septum interauricular
ecocardiograma de cabecera muestra una
e) La velocidad de conducción en el haz de
fracción de eyección (FE) severamente
deprimida, extensa anomalía en el movimiento his de 10 - 20 mm/seg en el sistema de
de la pared anterior e Insuficiencia Mitral leve. Purkinje 3000 - 4000 mm/seg, y en el
¿Cuál de los siguientes medicamentos debe miocardio ventricular 3000 - 5000
evitarse en mm/seg

esta paciente? COMENTARIO: El impulso corre a partir de aquí


por un haz penetrante del nodo AV,
a) Aspirina
generando otro retraso de aproximadamente
b) Inhibidor de GlicoProteína 2b/3a 0.04 segundos.

c) Heparina no fraccionada

d) Metoprolol Paciente varón de 47 años, fumador, con


antecedentes de hipercolesterolemia y diabetes
e) Estatinas en dosis altas
(con manifestaciones tardías, amputación
supracondilea de pierna izquierda hace 1 año).
Consulta por presentar en las últimas 48 horas
intensos episodios de dolor torácico opresivo

retroesternal, irradiados a hombros, de unos 15


minutos de duración. En el Holter que se le
realizó muestra, durante una de las crisis, un
descenso transitorio del segmento ST de
PASITO Nº3 HNHU
1.5 mm en V2 - V5 que se recupera en los
En relación al sistema de conducción cardiaca, episodios sin dolor. Se realizan exámenes
marque la afirmación correcta: auxiliares, no presenta elevación de
biomarcadores cardiacos y el ecocardiograma
a) El nodo sinusal (NSA) es el marcapaso
es normal. El diagnóstico es:
cardiaco por su menor frecuencia de
despolarización espontánea; está a) Angina estable
localizado en la unión del subepicardio
b) Pericarditis aguda
de la aurícula derecha con la pared
izquierda de la desembocadura de la c) Infarto agudo de miocardio con ST
vena cava inferior. elevado

b) En la unión auriculoventricular el d) Angina inestable


estímulo cardiaco experimenta un
retardo fisiológico de 0.06 - 0.10 seg. e) Infarto agudo de miocardio con ST
que permite un mayor llenado deprimido
ventricular antes de su sístole
COMENTARIO: Angina inestable con ST normal, COMENTARIO: BB (carvedilol, bisoprolol,
infradesnivelado no presenta elevación de metoprolol que es más cardioselectivo) se usa
biomarcadores cardiacos para disminuir FC, disminuir el consumo de O2 y
así ↓ y limitar el infarto al corazón.

BB reducen la PA
Mujer de 76 años con historia de insuficiencia
cardíaca por cardiopatía hipertensiva en
fibrilación auricular crónica que seguía
tratamiento con enalapril, digoxina, furosemida y Usted tiene un paciente en cuyo EKG se
warfarina. Consulta por presentar en la última encuentra QRS = 0.14 seg de duración con
semana náuseas e incremento de la disnea. La morfología rSR^ con R ancha en V1 y qRS con S
exploración muestra TA de 130/80 mmHg pulso ancho en V6. Qué diagnóstico de los
arterial de 116 lpm rítmico; en la auscultación
pulmonar se oyen crepitantes en la bases y en la siguientes plantearía en base al EKG
auscultación cardíaca refuerzo del segundo
a) Hemibloqueo anterior izquierdo
tono. El ECG muestra unataquicardia rítmica de
QRS estrecho a 116 b) Bloqueo completo de rama izquierda
lpm. ¿Qué actitud entre las siguientes es la más c) Hemibloqueo posterior izquierdo
adecuada?
d) Síndrome de preexcitación de tipo Wolff
a) Realizar monitorización de Holter Parkinson white
b) Suspender anticoagulantes orales e) Bloqueo completo de rama derecha
c) Asociar propranolol

d) Solicitar niveles de digoxina Paciente varón de 75 años que acude por


malestar precordial. Se le toma un EKG, una
e) Asociar amiodarona
lesión subepicárdica anteroseptal se
identificaría en su EKG como:
Una mujer de 72 años se presenta con disnea,
a) Ascenso de ST de V1 a V3
presión arterial de 95/50 mmHg, elevación del
segmento ST en V2 hasta V5 y edema pulmonar b) Ascenso de ST de V3 a V6
bilateral en la radiografía de tórax. Presenta
mayor hipotensión y luego obnubilación c) Descenso de ST de V1 a V3
requiriendo intubación. Se le prepara para PCI d) Ascenso de ST en I y aVL
emergente. Un ecocardiograma de cabecera
,muestra una fracción de eyección (FE) e) Descenso de ST en II, III, aVF
severamente deprimida, extensa anomalía en el
COMENTARIO:
movimiento de la pared anterior e insuficiencia
mitral leve ¿Cuál de los siguientes medicamentos ● Subendocárdica → descenso
debe evitarse en esta paciente? ● Subepicárdico → ascenso
a) Inhibidor de glicoproteína 2b/3a

b) Heparina no fraccionada El EKG mostrado es compatible con:


c) Estatinas en dosis bajas

d) Metoprolol

e) Aspirina
pulmonar y el estudio de función tiroidea una T4
libre elevada con una TSH indetectable ¿Cuál de
los fármacos utilizados puede ser el responsable
del cuadro que actualmente, presenta la
paciente?

a) Digoxina

a) Bloqueo completo de rama derecha del b) Captopril


haz de hiss
c) Amiodarona
b) Injuria subepicárdica
d) Propafenona
c) Taquiarritmia supraventricular
e) Diltiazem
d) Crecimiento del VD
COMENTARIO:
e) Bloqueo completo de rama izquierda del
Amiodarona: • Fibrosis pulmonar. • Alteraciones
haz de hiss
tiroideas (en su estructura lleva yodo:
COMENTARIO:
puede producir tanto hipo como hipertiroidismo)
PRÁCTICA (MIR). • Piel azulada. • Hepatitis tóxica. •

-Onda P0.8 - 0.11 sg Depósitos corneales. • Disminución del


crecimiento.
-Intervalo PR0.12 - 020 seg

-Complejo QRS < 0.10 seg


¿Cuál de los siguientes agentes ha demostrado
-Segmento ST <0.12 máximo (3 cuadraditos en ensayos clínicos randomizados -
aprox) prospectivos disminuir significativamente
-Onda T <0.20 seg (4 cuadraditos aprox) eventos cardiovasculares?

-Intervalo QT <0.38 pero es relativo porque varía a) Terapia con estatinas


con la FC b) Inhibidores de la SGTL2

c) Inhibidores de PCSK9
Mujer de 78 años diagnosticada de cardiopatía d) Agonistas de GLP1
hipertensiva con función sistólica conservada,
que en los últimos 2 años ha tenido 3 episodios e) Todos los mencionados
de fibrilación paroxística cardiovertidos
eléctricamente. Durante este tiempo ha recibido
diversos tratamientos que incluían algunos de AYALA
los siguientes fármacos: propafenona,
Paciente de 50 años con antecedentes de
amiodarona, digoxina, diltiazem y captopril.
tabaquismo, dislipidemia y diabetes consulta
Actualmente consulta por un cuadro de 2 meses
por presentar desde hace 36 horas episodios
de evolución de debilidad general y apatía,
repetidos de dolor torácico opresivo con
añadiendose en la última semana disnea
sudoración. El último y más prolongado tuvo una
progresiva hasta ser de pequeños
duración de 45 min.El
esfuerzos. El EKG muestra fibrilación auricular
ECG muestra elevación persistente del ST de 2
con frecuencia ventricular a 130 lpm, la RX de
mm en derivaciones DII, DIII y aVF. Señale la
tórax cardiomegalia con signos de congestión
afirmación más correcta, el tratamiento inicial auscultación pulmonar normal. ¿Cuál de los
debe incluir (salvo contraindicación) siguientes es el diagnóstico más probable?

a) únicamente aspirina a) ritmo idioventricular acelerado

b) aspirina, clopidogrel y aspirina b) síndrome de Dressler

c) nifedipino, aspirina y clopidogrel c) rotura de un músculo papilar

d) fibrinolisis, betabloqueantes y aspirina d) aneurisma gigante ventricular

e) aspirina, clopidogrel, heparina y e) infarto de ventrículo derecho


betabloqueantes

Una mujer de 72 años acude a urgencias con un


dolor torácico sugestivo de isquemia miocárdica
de 4 horas de evolución. En el ECG se observa un
descenso del segmento ST de 2 mm en V2-V6.
¿Cuál de las siguientes opciones terapéuticas no
es adecuada?

a) enoxaparina

b) clopidogrel
¿Cual es la terapia de primera opción en
c) ácido acetilsalicílico arritmias ventriculares causadas por isquemia
d) activador tisular del plasminógeno (t-PA) miocárdica aguda?

e) heparina sódica a) lidocaína

En relación con el empleo de la trombolisis en el b) procainamida


infarto agudo de miocardio, señale la c) quinidina
afirmación falsa d) propranolol
a) Es útil para disminuir el tamaño de la e) adenosina
zona infartada

b) Es útil para disminuir las arritmias

c) Es útil para disminuir la mortalidad

d) Es útil para limitar la disfunción


ventricular izquierda

e) Solo es útil en las primeras horas


postinfarto

Paciente varón de 46 años que ingresa con


intenso dolor precordial y elevación persistente
del segmento ST en derivaciones II, III y aVF,se
administra tratamiento trombolítico y en las
horas siguientes presenta distensión venoso
yugular, signo de Kussmaul, hepatomegalia,
EXAMEN HIPÓLITO:
tensión arterial sistólica de 70 mmHg y
1. El presente ECG fue tomado en betabloqueadores. De las siguientes
urgencias en un varón de 67 años con situaciones, cuál no corresponde a una
factores de riesgo CV, quien sufrió una contraindicación absoluta para el uso
parada cardiorrespiratoria de betabloqueadores?
extrahospitalaria que fue reanimado
a. bloqueo AV avanzado
con éxito. Se encuentra lo siguiente.
Marque la alternativa correcta: b. Enfermedad pulmonar crónica

c. Bradicardia sintomática

d. Broncoespasmo

e. Insuficiencia cardiaca
inestable

a. Ritmo No sinusal, FC 75 lpm,


Hemibloqueo posterior
derecho, supradesnivel del ST
en V4 y aplanamiento de onda
T en aVL

b. Ritmo no sinusal, FC 75 lpm, 3. En los pacientes que presentan shock se


Hemibloqueo posterior verifica un descenso marcado de la
derecho, descenso del ST en V1 presión arterial sistémica. En relación al
y V2 aplanamiento en onda T en estado de shock, señale la alternativa
aVL correcta

c. Ritmo sinusal, FC 75 lpm, a. En el shock cardiogénico, el


Hemibloqueo anterior gasto cardiaco está
izquierdo, descenso del ST de disminuidos y las resistencias
hasta 1 mm en V4 y V5? y vasculares se hallan
aplanamiento de onda T en aumentadas
aVL* b. En el shock de origen
d. Ritmo sinusal FC 100 lpm, hemorrágico, el gasto cardiaco
bloqueo completo de rama está elevado y las resistencias
izquierda, infradesnivel del ST vasculares están
de V4 a V5? y aplanamiento de incrementadas
onda T en aVR c. En el shock de origen séptico,
e. Ritmo sinusal FC 50 lpm, las resistencias vasculares y el
bloqueo de rama derecha, gasto cardiaco se encuentran
infradesnivel del ST en cara incrementados
inferior y aplanamiento de d. En el shock producido por
onda T en aVL insuficiencia hepática, la
2. Paciente mujer de 50 años con resistencia vascular sistémica
insuficiencia cardíaca por cardiopatía se halla incrementada y el
isquémica crónica postinfarto y gasto cardiaco reducido
disfunción diastólica. Se decide utilizar
e. En el shock que es causado por
crisis tirotóxica, las resistencias Flujo pulmonar Flujo pulmonar
vasculares se hallan normal o aumentado
incrementadas y el gasto disminuido
cardiaco está reducido

-Estenosis -Comunicación
Pulmonar interauricular

-Coartación aórtica -Comunicación


interventricular
-Estenosis aórtica
congénita -Ducto arterial
persistente
4. Paciente varón de 11 años. quien nunca
-Canal
ha presentado cianosis. La madre
auriculoventricular
refiere que le diagnosticaron de soplo
común
cardíaco desde el nacimiento. En las
últimas semanas ha desarrollado
disnea de esfuerzo. En el examen físico
se ausculta un ruidoso soplo sistólico en Cardiopatía Cianótica
el segundo espacio intercostal
izquierdo, además hay desdoblamiento
amplio del segundo ruido, no fijo. El
componente pulmonar del segundo Flujo pulmonar Flujo pulmonar
ruido cardiaco tiene intensidad normal o aumentado
disminuida. En el ECG se encuentra disminuido
hipertrofia ventricular derecha. Con lo
anterior, el diagnóstico más probable
es. -Tetralogía de Fallot -Transposición de
grandes vasos
a. Ductus arterioso persistente
-Anomalía de
b. Defecto del septum Ebstein -Drenaje Venoso
interventricular anómalo total
-Atresia tricuspidea
c. Estenosis pulmonar -Síndrome del
ventrículo izquierdo
d. Defecto del septum
hipoplásico
interauricular

e. Estenosis aórtica -Troncus

-Ventrículo único

Cardiopatía Acianótica
C) Pericardiectomía quirúrgica
programada

D) Pericardiocentesis eco guiada

ANSWER: D

5. Paciente varón de 70 años, refiere que Se define como pericarditis incesante cuando:
desde hace un año aproximadamente
presenta disnea de esfuerzo, angina de A) Dura más de 4 - 6 semanas sin remisión
pecho y además ha tenido ingresos por B) Dura más de 1 año con remisión
emergencia por síncope en 2
oportunidades. En el examen físico se C) Duran menos de 4 semanas sin
ausculta un soplo sistólico en el remisión
segundo espacio intercostal derecho.
D) Dura menos de 2 semanas sin remisión
Ante ello, el diagnóstico más probable
es: ANSWER: A

a. Insuficiencia tricuspídea Varón de 55 años con HTA mal controlada como


antecedente, es diagnosticado con infarto de
b. Insuficiencia aórtica (soplo miocardio STE anterior extenso de 3 horas de
diastólico) evolución. El médico de guardia solicita
c. Estenosis mitral cateterismo cardiaco izquierdo (ICP primaria),
pero le informan que no se cuenta con dicho
d. insuficiencia mitral procedimiento hasta en 5 horas a más. ¿Qué
terapia instauraría inmediatamente?
e. Estenosis aórtica
A) Morfina horario y diclofenaco para el
dolor

B) Nitrato oral y anticoagulantes es


suficiente

C) Antiagregantes, anticoagulante e iniciar


la fibrinólisis

D) Atorvastatina y Espironolactona a dosis


BANCO TALLERES H2M plenas para calmar el dolor

Varón de 50 años con diagnóstico de ANSWER: C


pericarditis aguda efusiva, presenta deterioro
DIAGNÓSTICO:
progresivo de los síntomas con ortopnea y
edemas de miembros inferiores. Es evaluado en A) Infarto de miocardio LATERAL
emergencia donde se describe ausencia de
B) Infarto de miocardio INFEROPOSTERIOR
colapso venoso yugular en inspiración e
ingurgitación presente de forma bilateral. Se le C) Infarto de miocardio INFERIOR
realiza un Ecocardiograma de emergencia con POSTERO LATERAL
evidencia de efusión pericárdica severa y
colapso diastólico del ventrículo derecho. Diga D) Infarto de miocardio ANTEROLATERAL
usted qué tratamiento indicaría. ANSWER: C
A) Colchicina + aines orales + warfarina DIAGNÓSTICO:
B) Aines + warfarina A) Infarto de miocardio STE septal
B) Infarto de miocardio STE ANTERIOR insuficiencia cardiaca. Qué criterios
electrocardiográficos debería buscar para
C) Infarto de miocardio STE POSTERIOR confirmar la presencia de bloqueo completo de
D) Infarto de miocardio STE rama derecha en este paciente. Marque la
INFERO-LATERAL correcta.

ANSWER: C A) QRS mayor o igual a 0.12 segundos

DIAGNÓSTICO: B) V1 morfología de rsR’ (mellada)

A) Infarto de miocardio STE SEPTAL C) I y V6 : predominio de la onda R y onda


T positiva
B) Infarto de miocardio STE ANTERIOR
D) Todas las anteriores
C) Infarto de miocardio STE ANTERIOR
EXTENSO ANSWER: D

D) Infarto de miocardio STE DE TRONCO Varón de 25 años con sospecha de


DE CORONARIA IZQUIERDA miocardiopatía hipertrófica, por presencia de
impulso ventricular izquierdo con
ANSWER: C desplazamiento izquierdo. Se solicita un EKG
¿Diga usted que criterios electrocardiográficos
DIAGNÓSTICO:
ameritaría evaluar?
A) Infarto de miocardio STE INFERIOR
A) Criterios de Vereckei (taquicardia)
B) Taponamiento cardiaco
B) Criterios de Brugada(taquicardia)
C) Pericarditis aguda
C) Criterio de Sokolow-Lyon
D) infarto de miocardio STE
D) Todas las anteriores
ANTEROSEPTAL
ANSWER: C
ANSWER: C

Ecocardiografía vista en eje largo, se observa el


ventrículo izquierdo (VI), aurícula izquierda (AI) y
aorta (Ao), ¿Qué es lo más llamativo?

A) Derrame pericárdico concéntrico

B) Comunicación interventricular

C) Hipertrofia Ventricular izquierda


concéntrica

D) Prolapso de la válvula mitral por


compromiso del festón P2 EKG
ANSWER: C EKG 6
Mujer de 35 años con disnea CF III, edemas de
miembros inferiores y soplo sistólico IV/VI a nivel
paraesternal derecho con desdoblamiento fijo
del segundo ruido cardiaco. Se plantea como
sospecha una comunicación interauricular e
Diagnóstico:

a. Bloqueo AV completo + IMA anterior Diagnostico:

b. Bloqueo AV completo + IMA septal a. Taquicardia + bloqueo de rama


izquierda
c. Bloqueo AV 2° tipo II + IMA anterior
b. Fibrilación ventricular
d. Bloqueo AV 2° tipo I + IMA anterior
c. Taquicardia con bloqueo de rama
derecha
EKG 7 d. Taquicardia ventricular

EKG 10

Diagnóstico:

a. Hipertrofia ventricular derecha Diagnóstico:

b. Bloqueo completo de rama izquierda a. Hipertrofia ventricular izquierda

c. Hemibloqueo posterior izquierdo b. IMA antero septal

d. Bloqueo completo de rama derecha c. Bloqueo completo de rama derecha

EKG 8 d. Bloqueo completo de rama izquierda

EKG 1

EKG 9
Diagnostico:
a. infarto miocardio LATERAL

b. infarto miocardio INFEROPOSTERIOR EKG 4

c. infarto miocardio INFERO POSTERO LATERAL

d. infarto miocardio ANTEROLATERAL

EKG 2

Diagnostico:
Diagnostico:
a. Infarto de miocardio STE septal.
a. Infarto de miocardio STE septal.
b. Infarto de miocardio STE ANTERIOR
b. Infarto de miocardio STE INFERIOR
c. Infarto de miocardio STE POSTERIOR
c. Infarto de miocardio STE INFERO POSTERIOR
d. Infarto de miocardio STE INFERO-LATERAL
d. Infarto de miocardio STE INFERO POSTERO
EKG 3 LATERAL

EKG 5

Diagnostico:

a. Fibrilación atrial paroxística Diagnostico:


b. Taquiarritmia QRS ancho probable a. Fibrilación auricular
Taquicardia ventricular
b. Taquicardia auricular multifocal.
c. Taquiarritmia QRS angosto probable
taquicardia de reentrada nodal c. Taquicardia ventricular lenta

d. Taquiarritmia QRS angosto probable d. Flutter auricular típico


taquicardia tipo Wolf Parkinson White.
PRÁCTICA 2022-1 B. Insuficiencia mitral
C. Insuficiencia aórtica
Paciente mujer de 60 años, con insuficiencia
D. Estenosis mitral
cardíaca por cardiopatía isquémica crónica post
E. Estenosis aórtica
infarto y disfunción diastólica. Se decide utilizar
betabloqueadores. De las siguientes situaciones Paciente varón de 75 años que acude por
¿Cuál no corresponde a una contraindicación malestar precordial. Se le toma un EKG, una
absoluta para el uso de betabloqueantes ? lesión subepicárdica anteroseptal se
identificaría en su EKG como:
A. Bradicardia sintomática
B. broncoespasmo A. Descenso de ST en I, III y aVF
C. Insuficiencia cardiaca inestable B. Ascenso de ST de V1 a v3
D. Enfermedad pulmonar crónica C. Descenso de ST de V1 a v3
E. Bloquea Av avanzado D. Ascenso de ST en l y aVL
E. Ascenso de ST de v3 a V6
Usted tiene un paciente en cuyo ECG se
encuentra QRS=0,14 seg de duración con
morfología rSR con R ancha en VI y qRS con S
ancha en V6. Qué diagnóstico de los siguientes
plantería en base al EKG

A. Hemibloqueo anterior izquierdo


B. Sindrome de preexcitacion de tipo Wolf
Parkinson White
C. Hemibloqueo posterior izquierdo
D. Bloqueo completo de rama derecha
E. Bloqueo completo de rama izquierda A. Bloqueo completo de rama izquierda del
Se atiende una paciente de 65 años con haz de Hiss
diagnóstico de insuficiencia cardíaca por lo cual B. Taquiarritmia supraventricular
recibe tratamiento con digoxina y diuréticos tipo C. Injuria subepicárdica
tiazídicos. Ante ello, señale cuál de los siguientes D. Bloqueo completo de rama derecha del
fármacos o situaciones puede aumentar el haz de Hiss
riesgo de toxicidad de la digoxina? E. Crecimiento de ventrículo derecho

A. Administración concomitante de Son causas de supradesnivel del segmento ST


captopril en el EKG :(marque la alternativa correcta)
B. Hipocalcemia A. Pericarditis aguda, hipotermia, injuria
C. Hipermagnesemia subepicárdica, hipokalemia
D. Potasio sérico de 2,5 mEq/L hiperkaliúrica.
E. Administración conjunta de hidralazina B. Marcapaso, miocardiopatías,
Paciente varón de 70 años, refiere que desde hipokalemia, sobredosis digitálica,
hace un año aproximadamente presenta disnea injuria subendocárdica.
de esfuerzo, angina de pecho y además ha C. Hiperkalemia, miocardiopatías, injuria
tenido ingresos por emergencia por síncope en subendocárdica, pericarditis
2 oportunidades. En el examen físico se ausculta epistenocárdica.
un soplo sistólico en el segundo espacio D. Injuria subepicárdica, hipotermia,
intercostal derecho. Ante ello, el diagnóstico más hiperkalemia, pericarditis aguda.
probable es: E. Hipokalemia, intoxicación digitálica,
marcapasos migratorio, fenómeno de la
A. Insuficiencia tricuspidea concertina
Paciente varón de ll años, quien nunca ha bradicinina por lo que se asocian
presentado cianosis. La madre refiere que le frecuentemente a tos seca
diagnosticaron de soplo cardiaco desde el D. Los antagonistas del receptor de
nacimiento. En las últimas semanas ha aldosterona rara vez producen
desarrollado disnea de esfuerzo. En el examen hiperpotasemia y se pueden usar si la
físico se ausculta un ruido soplo sistólico en el creatinina es mayor de 2,5mg/dl
segundo espacio intercostal izquierdo, además E. Los IECAS reducen la vasoconstricción,
hay desdoblamiento amplio del segundo ruido, producen aumento del Na sérico y
no fijo. El componente pulmonar del segundo reducen los niveles de K sérico
ruido cardiaco tiene intensidad disminuida. En
el EKG se encuentra hipertrofia ventricular
derecha. Con lo anterior, el diagnóstico más
probable es:

A. Estenosis pulmonar
B. Estenosis aórtica
C. Defecto del septum interauricular
D. Ductus arterioso persistente
E. Defecto del septum interventricular

En relación de score de GRACE en siCA Sin


elevación del sT, marque la alternativa correcta

A. Para su cálculo no se consideran como


factores la creatinina sérica ni la
frecuencia cardiaca en reposo
B. No se considera en la evaluación al alta
la presencia de infarto de miocardio
antigüo
C. El score no permite calcular el riesgo de
muerte a los 6 meses según la
puntuación total al alta
D. Permite calcular el riesgo de muerte
intrahospitalaria en pacientes
sometidos a coronariografía previa a la
hospitalización
E. Un factor considerado en el score es la
presencia de paro cardiaco al ingreso

Con respecto a la insuficiencia cardiaca, marque


lo correcto:

A. Los bloqueantes de canales de calcio


han demostrado disminuir la mortalidad
en la Ins. Cardiaca con FE reducida.
B. La digoxina reduce el número de
ingresos hospitalarios por Ins. Cardíaca
sin mejorar la mortalidad global
C. Los bloqueadores del receptor de
angiotensina ll aumentan los nivel de

You might also like